Practice questions Flashcards
A 43-year-old man presents with a painless lump in his groin that appeared 2 weeks ago. He claims that the lump protrudes on defecation and he also reports becoming constipated recently. On examination, the lump is reduced, and a finger is placed over the midpoint of the inguinal ligament. When the patient is asked to cough, the lump reappears. What is the most likely diagnosis? A Direct inguinal hernia B Indirect inguinal hernia C Femoral hernia D Obturator hernia E Spigelian hernia
A
Which of the following murmurs is associated with severe aortic regurgitation? A Austin-Flint B Graham-Steell C Gibson D Carey-Coombs E Barlow
A
A 67-year-old man is brought into A&E having been involved in a road traffic accident. On examination, he opens his eyes to pain, makes a few grunting noises and withdraws his legs from painful stimuli. What is his GCS? A2 B4 C6 D8 E 10
D
A 2-year-old boy is brought to the GP after his father noticed some swelling around his eyes. On examination, there is periorbital and pedal oedema. A urine dipstick is positive for proteins and negative for blood. What is the most likely diagnosis?
A IgA nephropathy
B Membranous glomerulonephritis
C Rapidly progressive glomerulonephritis
D Minimal change glomerulonephritis
E Henoch-Schӧnlein purpura
D
A 54-year-old man is brought into A&E with a suspected acute coronary syndrome. An ECG is performed, which reveals ST elevation in leads I, aVL, V5 and V6. Which coronary artery has been occluded?
A Left main stem
B Left anterior descending coronary artery
C Left circumflex coronary artery
D Right coronary artery
E Posterior descending artery
C
A 46-year-old housewife visits her GP complaining of pain in the joints of her hands that has gradually got worse over 3 months. It has started affecting her ability to complete daily tasks such as cooking for her children. Both of her hands are affected equally and the pain and stiffness is worst in the morning but gets better when she starts using her hands. On closer inspection, her hands do not appear to be deformed although her metacarpophalangeal joints and proximal interphalangeal joints appear slightly swollen, warm and tender. What is the most likely diagnosis? A Reactive arthritis B Osteoarthritis C Rheumatoid arthritis D Psoriatic arthritis E Septic arthritis
C
A 56-year-old man with a history of alcoholism complains of intermittent epigastric pain that radiates through to his back. When questioned, he admits to losing about 3 kg in weight over the past 6 months and says that his stools have become pale and difficult to flush away. Which investigation would you request to aid the diagnosis? A Serum amylase B Blood cultures C Faecal elastase D CA 19-9 E OGD
C
A 25-year-old female has suffered from shortness of breath over the past 2 months. She gets particularly breathless when she exerts herself, and has had to stop going on her morning jog. She has not experienced a cough, fever or chest pain. She has no past medical history of note, however, her periods have become quite heavy over the past 3 or 4 months. What is the most likely diagnosis? A Hyperthyroidism B Anaemia C Pneumonia D COPD E Asthma
B
A 46-year-old man, with a history of type 1 diabetes, visits the GP for an HbA1c reading. He has recently been feeling more tired than usual and has noticed that the skin on his hands has become darker over the past few months. On examination, hepatomegaly and a tanned complexion (despite not having been on any recent holidays) are noted. Haemochromatosis is suspected and iron studies are requested. Which set of results would be consistent with haemochromatosis?
A High serum iron, high ferritin, high transferrin, low transferrin saturation, low TIBC
B High serum iron, low ferritin, low transferrin, high transferrin saturation, low TIBC
C High serum iron, high ferritin, high transferrin, high transferrin saturation, low TIBC
D High serum iron, high ferritin, low transferrin, high transferrin saturation, high TIBC
E High serum iron, high ferritin, low transferrin, high transferrin saturation, low TIBC
E
10. A 72-year-old woman has recently suffered a fracture of her right distal radius after falling on an outstretched hand. She is at high risk of osteoporosis because she is post-menopausal and has undergone several decades of steroid treatment for her asthma. A DEXA scan is performed. Which result would be diagnostic of osteoporosis? A T score of -1.5 or worse B T-score of -2 or worse C T-score of -2.5 of worse D T-score of -3 or worse E T-score of -3.5 or worse
C
A 52-year-old man was watching TV yesterday when he suddenly become very aware of his heart beating rapidly. This lasted around 45 mins and then subsided spontaneously. It has happened several times over the past 2 months. An ECG reveals no abnormalities. However, due to the strong suspicion of atrial fibrillation, the patient is placed on a 24-hr tape, which confirms the diagnosis. Which scoring system should be used to determine the benefit of long-term anticoagulation in this patient? A QRISK2 score B ABCD2 Score C GRACE score D CHA2DS2-VASc score E CURB-65 score
D
Which of the following is not a feature of background diabetic retinopathy? A Hard exudates B Soft exudates C Microaneurysms D Blot haemorrhages E Leakage of lipids from blood vessels
B
A 53-year-old man, who has recently recovered from a diarrhoeal illness, comes to A&E with a 1-week history of gradually worsening weakness in his legs. A neurological examination reveals reduced tone, reduced reflexes and impaired sensation in both lower limbs. He adds that the weakness began in his feet and has gradually progressed up his legs. Guillain-Barré syndrome is suspected. Which of the following parameters should be closely monitored in this patient? A Body temperature B Serum osmolality C Serum potassium D Forced vital capacity E Urine output
D
A 14-year-old school boy was diagnosed with asthma 6 months ago. He was
given a salbutamol inhaler to use PRN, however, he continued to have regular
episodes of breathlessness. He was started on a regular inhaled corticosteroid
(beclomethasone) 3 months ago. Although the frequency of his attacks has reduced
with the medication, he is still experiencing bouts of breathlessness about 4 times
per week. What would be the next most appropriate step in his management?
A Increase the dose of salbutamol
B Add an inhaled corticosteroid
C Add an oral corticosteroid
D Add a long-acting beta agonist
E Montelukast therapy
B
Which of the following is the most common cause of chronic kidney disease? A Hypertension B Diabetes mellitus C Glomerulonephritis D Pyelonephritis E Polycystic kidney disease
B
A 46-year-old woman has been suffering from frequent headaches over the past 4 months, along with some blurring of vision. She also mentions that she has developed a fiercely itchy rash on several occasions, usually occurring soon after she has had a bath. What is the most likely diagnosis? A Anaemia B Acute lymphoblastic leukaemia C Chronic myeloid leukaemia D Polycythaemia vera E Myelofibrosis
D
A 27-year-old female comes to A&E complaining of severe right iliac fossa pain. She has vomited three times whilst waiting to be seen. On examination, there is rebound and percussion tenderness in the right iliac fossa and the pain gets worse
when the doctor extends the patient’s hip. What is the name of the sign being elicited?
A Murphy’s sign
B Cope sign
C Rovsing’s sign
D Psoas sign
E Aarons sign
D
A 74-year-old man presents with a skin lesion on his left cheek, which has gradually grown over 4 months. On closer examination, the lesion has raised, everted edges with an ulcerated centre revealing a keratotic core. What is the most likely diagnosis? A Basal cell carcinoma B Squamous cell carcinoma C Melanoma D Keratoacanthoma E Actinic keratosis
B
A 53-year-old man presents to A&E with severe pain in his right flank that radiates to his right groin. Ureteric colic is suspected and a CT-KUB is requested. The CT-KUB confirms the diagnosis but it also shows an abdominal aortic aneurysm with a diameter of 4.7 cm. When questioned, the patient denies any back pain (other than the pain caused by ureteric colic) or symptoms of vascular disease. What is the most appropriate management option for this patient?
A Reassure and discharge
B Surveillance with an ultrasound scan every 1 year
C Surveillance with an ultrasound scan every 6 months
D Surveillance with an ultrasound scan every 3 months
E Surgical repair of the aneurysm
D
A 38-year-old man is complaining of excessive thirst and frequent urination over the past month. He describes the thirst as being ‘insatiable’, and claims to drink about 8-12 litres of water every day. He undergoes a water deprivation test, which revealed the following results:
Urine osmolality
2 hr : 212 mOsm/kg 4 hr: 227 mOsm/kg
6 hr : 221 mOsm/kg 8 hr: 242 mOsm/kg
Following administration of 2 mg IM desmopressin (DDAVP) : 278 mOsm/kg (normal
> 600 mOsm/kg).
What is the most likely diagnosis?
A Diabetes mellitus type 1 B Central diabetes insipidus C Nephrogenic diabetes insipidus D Psychogenic polydipsia E Hyperparathyroidism
C
A 37-year-old man visits the GP because his partner has recently noticed some dark patches on his back. On examination, there are multiple painless red papules on his back, some of which have merged to form purple plaques. He was diagnosed with HIV 8 years ago. What is the most likely diagnosis? A Discoid lupus B Guttate psoriasis C Kaposi’s sarcoma D Shingles E Chronic mucocutaneous candidiasis
C
A 16-year-old school girl has been suffering from depression for 6 months. Last night (12 hours ago) she decided to end her life and ingested 40 paracetamol tablets. She has since decided that she has made a mistake and does not want to end her life. She appears reasonably well when she presents to A&E, only complaining of some mild nausea. What is the most appropriate treatment option? A N-acetylcysteine B Naloxone C Flumezanil D Atropine E Sodium bicarbonate
A
A 7-year-old girl has had severe, bloody diarrhoea with cramping abdominal pain for the past 3 days. She has also been emptying her bladder less and less frequently. Blood tests, including a blood film, are performed which revealed a low
Hb, low platelets and the presence of schistocytes. U&Es are also performed: Creatinine : 182 micromol/L (baseline = 92 micromol/L)
Urea : 9.2 mmol/L (2.5-6.7)
A stool culture identifies E. coli O157. What is the most likely diagnosis?
A Microangiopathic haemolytic anaemia B Haemolytic uraemic syndrome C Thrombotic thrombocytopaenic purpura D Disseminated intravascular coagulation E Gastroenteritis
B
A 37-year-old plumber presents to his GP with a week-long history of headache, diarrhoea and fever. In the last 2 days, he has also developed a dry cough and his wife has commented that he has appeared to be quite confused recently. What is the most appropriate investigation?
A CT head scan B Urinary antigens C Stool culture D LFTs E U&Es
B
Which system is used to stage prostate cancer? A Breslow thickness B Gleason staging C GRACE score D W ells score E Dukes’ staging
B
A 28-year-old girl presents with a 5-day history of urinary frequency, dysuria and mild suprapubic pain. Urine dipstick is positive for nitrites and leucocytes. A urinary tract infection is suspected and an MSU is taken and sent to the lab. Whilst awaiting culture and sensitivities, which empirical treatment option is best for this patient?
A Trimethoprim B Benzylpenicillin C Vancomycin D Metronidazole E Levofloxacin
A
A 68-year-old male visits his GP complaining of constipation, rectal bleeding and itchiness around his anus. He often feels ‘a lump’ hanging out after defecating which he has to push back in himself. On examination, anal tone is weak and a protruding mass is felt which has palpable muscular rings. What is the most likely diagnosis?
A Grade 3 haemorrhoids B Grade 4 haemorrhoids C Perianal abscess D Type 1 rectal prolapse E Type 2 rectal prolapse
E
A 54-year-old man is complaining of sharp, central chest pain that has arisen over the last 24 hours. On inspection, the patient is sitting forward on the examination couch. On auscultation, a scratching sound is heard – loudest over the lower left sternal edge, when the patient is leaning forward. He also has a low-grade fever. He has a past medical history of a ST-elevation MI which was diagnosed, and treated with PCI, 6 weeks ago. What is the most likely diagnosis? A Viral pericarditis B Constrictive pericarditis C Cardiac tamponade D Dressler syndrome E Tietze syndrome
D
A 66-year old man has been experiencing pain in his right calf for the last 6 months. Initially, the pain would come about whenever he went for a walk, and it would be relieved by rest. However, over the last few weeks he has experienced pain during rest. The pain is particularly bad at night, and he gets some relief from dangling his leg over the end of the bed. Recently, he has noticed a small, elliptical ulcer appear in between his toes on his right foot. He has a past medical history of ischaemic heart disease, and underwent a CABG 8 years ago. What is the most likely diagnosis? A Intermittent claudication B Critical limb ischaemia C Acute limb ischaemia D Leriche syndrome E Chronic deep vein thrombosis
B
A 3-year-old boy is brought, by his father, to see his GP because he has had a fever for the past 4 days and has cried in pain every time he has tried to eat. On examination, the patient’s gums look red and swollen and there are small vesicles and ulcerations along the gumline. The GP suspects gingivostomatitis. What is the most likely cause? A Varicella B Primary HSV1 infection C Reactivation of HSV1 D HSV2 infection E Infectious mononucleosis
B
A 31-year-old man presents to his GP with a 2-day history of central chest pain, which gets worse when he breathes in and when exercising. Cardiovascular and respiratory examinations detect no abnormalities, however, the patient winces in pain when the GP palpates for heaves and thrills. What is the most likely diagnosis? A Pulmonary embolism B Myocarditis C Tension pneumothorax D Costochondritis E Pleurisy
D
Which of the following is unlikely to cause pleuritic chest pain? A Tension pneumothorax B Rib fracture C Pulmonary fibrosis D Pneumonia E Pericarditis
C
A 28-year-old PhD student books an appointment to see her GP about some small lumps on her groin. On closer inspection, there are multiple small, firm, dome- shaped lumps with an umbilicated centre. On direct questioning, she reveals that she has recently had a new sexual partner. What is the most likely diagnosis? A Molluscum contagiosum B Varicella zoster C Syphilis D Gonorrhoea E Sebaceous cysts
A
A 71-year-old woman presents to A&E with a headache that has gradually been getting worse over the past week. The pain is localised over the left half of her forehead and does not radiate. She has also been eating less frequently as her jaw becomes painful when she chews her food. On direct questioning, she admits to experiencing some stiffness and pain in her shoulders over the past 6 months. On examination, she has a thickened, non-pulsatile temporal artery. What is the first step in her management? A Check ESR B Temporal artery biopsy C IV hydrocortisone D Oral prednisolone E IV antibiotics
D
A 24-year-old female, who has recently returned from a 3-week trip to Vietnam, complains that she has been feeling ‘under the weather’ with fevers and joint pain. On direct questioning, she reveals that she had unprotected sexual intercourse with a stranger whilst in Vietnam. She is jaundiced and has right upper quadrant tenderness. Hepatitis B serology is requested. The results are shown below: HBsAg + HBeAg - HBcAb IgM + HBcAb IgG + HBsAb - What is the hepatitis status of this patient? A Acute infection B Chronic infection C Cleared D Vaccinated E Susceptible
A
6. An 18-year-old female is brought to A&E, by ambulance, having been involved in a road traffic accident. She has bled significantly and needs an urgent blood transfusion. Her blood group is AB+. Which of the following blood groups will she be able to accept? A A+ B AB- C B- D O- E All of the above
E
Which clinical test can be used to diagnose ankylosing spondylitis? A Schober's test B Schirmir's test C Buerger's test D Weber's test E Tensilon test
A
A 53-year-old Afro-Caribbean man visits the GP to have his blood pressure measured. He has a history of hypertension and has been taking Amlodipine for 6 months. His blood pressure is 162/110 mm Hg. The GP is not satisfied with his blood pressure control and wants to step up his management. Which medication should be added? A Verapamil B Spironolactone C Bendroflumethiazide D Doxazosin E Enalapril
E
An 81-year-old man has been urinating about 12 times every day, including at night, and has difficulty starting a stream, which he describes as being ‘very weak’. He has also suffered from lower back pain over the past month. A DRE is performed, revealing an asymmetrically enlarged, nodular prostate gland. Which investigation is most likely to provide a definitive diagnosis? A PSA B Acid phosphatase C Transrectal ultrasound-guided biopsy D CT Scan E Isotope bone scan
C
- A 61-year-old man is brought to A&E by his daughter as he has become increasingly breathless over the past 24 hours and he has been coughing up a large amount of green sputum. He has a past medical history of COPD. Arterial blood gases are requested which show the following results (on room air):
pH : 7.33 (7.35-7.45)
PaO2 : 6.7 kPa (> 10.6 kPa on air)
PaCO2 : 9.6 kPa (4.7 - 6 kPa on air) HCO3- : 33 mmol/L (22 – 28 mmol/L) Respiratory Rate : 22 /min
What is the diagnosis?
A Partially compensated respiratory acidosis
B Fully compensated respiratory acidosis
C Partially compensated metabolic acidosis
D Fully compensated metabolic acidosis
E Acute type 1 respiratory failure
A
. A 73-year-old man has come to the outpatient clinic with his wife. She says that her husband seems very confused on some days and then seems completely normal on others. During the consultation, the patient appears confused with an AMTS of 4/10. He is distressed and claims that he can see little men running across the desk towards him. The doctor also notices a resting tremor. What is the most likely diagnosis? A Lewy body dementia B Alzheimer’s disease C Depressive pseudodementia D Frontotemporal dementia E Vascular dementia
A
Which of the following is part of the diagnostic criteria for diabetes mellitus?
A Two fasting blood glucose > 7.8 mmol/L in an asymptomatic patient
B One fasting blood glucose > 11.1 mmol/L in an asymptomatic patient
C One random blood glucose > 11.1 mmol/L in a symptomatic patient
D Two random blood glucose > 7 mmol/L in an asymptomatic patient
E Glycosuria and ketonuria on urine dipstick
C
The red reflex is an important part of the ophthalmological examination. Which
of the following conditions can result in loss of the red reflex?
A Herpes simplex keratitis
B Cataract
C Astigmatism
D Conjunctivitis
E Aniridia
B
A 32-year-old man presents to his GP with an 8-month history of diffuse abdominal pain and frequent loose motions. He has also been passing blood with his stools. On examination, a red ring around the cornea is seen in both eyes. The patient is referred for a colonoscopy and biopsy. What would you expect the biopsy to show?
A Non-caseating granulomas
B Eosinophilic infiltration
C Villous atrophy and crypt hyperplasia
D High grade dysplasia and metaplastic columnar epithelium
E Mucosal ulcers, goblet cell depletion and crypt abscesses
E
A 28-year-old professional cyclist visits his GP complaining of headaches and blurred vision. He is worried that his symptoms will affect his performance in an important race in 3 weeks’ time. On direct questioning, he admits to taking ‘performance enhancers’ in preparation for his race. On examination, scratch marks are seen on his trunk. What is the most likely diagnosis? A Thalassaemia B Polycythaemia rubra vera C Secondary polycythaemia D Hodgkin’s lymphoma E Non-Hodgkin’s lymphoma
C
A 22-year-old teacher visits her GP after fainting several times over the past 2 months. She does not experience any palpitations, light-headedness or auras before she faints, and she recovers very quickly. She has not bitten her tongue or become incontinent at any point. When questioned about the timing of these episodes, she reveals that she has only ever collapsed at work after she has been writing on the whiteboard for quite some time. On examination, a firm, immobile lump is palpated in her left supraclavicular fossa. What is the most likely diagnosis? A Paroxysmal atrial fibrillation B Transient ischaemic attack C Atonic seizures D Subclavian steal syndrome E Vasovagal syncope
D
Which of the following triads best describes the main features of nephrotic syndrome?
A Proteinuria, Hypoalbuminaemia, Oedema
B Haematuria, Hypoalbuminaemia, Oedema
C Proteinuria, Haematuria, Hyperlipidaemia
D Proteinuria, Haematuria, Hypoalbuminaemia
E Frequency, Urgency, Dysuria
A
A 63-year-old man with ascending bilateral limb weakness and ascending paraesthesia is diagnosed with Guillain-Barré syndrome. 3 weeks prior to the onset of these symptoms he suffered from gastroenteritis. Which organism is most likely to have caused this infection? A Salmonella B Campylobacter jejuni C E. coli 0157 D Rotavirus E Entamoeba histolytica
B
A 79-year-old care home resident is admitted to hospital with a 4-day history of a cough productive of green sputum. She has also experienced some chest pain and shortness of breath. A chest X-ray shows an area of consolidation in the right middle lobe with a right-sided pleural effusion. What is the most appropriate treatment option? A Co-amoxiclav and clarithromycin B Co-trimoxazole C Metronidazole D Flucloxacillin E Rifampicin and isoniazid
A
A 76-year-old care home resident has fractured his neck of femur having fallen out of bed. He is referred to the orthopaedic surgery department and undergoes an operation. Post-operatively, he is in considerable pain and is given 5 mg morphine sulphate. Which of these side-effects is he most likely to experience? A Constipation B Blurred vision C Cough D Tremor E Rash
A
A 65-year-old man, who is currently undergoing treatment for chronic lymphocytic leukaemia, presents with an extremely painful left great toe. On closer inspection, he has a fiercely inflamed left metatarsophalangeal joint. He has no other symptoms. What would you expect to see on analysis of the joint fluid aspirate?
A High WCC, turbid fluid
B Positively birefringent, rhomboid-shaped crystals
C Positively birefringent, needle-shaped crystals
D Negatively birefringent, rhomboid-shaped crystals
E Negatively birefringent, needle-shaped crystals
E
A 47-year-old woman has had several ‘dizzy spells’ over the past 6 weeks. She has been feeling very faint when getting out of bed in the morning and has also experienced some vague abdominal pain along with weight loss and lethargy. Examination reveals dark palmar creases and vitiligo on her back. What is the most appropriate investigation to request? A Full blood count B Fasting blood glucose C ECG D Short synacthen test E Thyroid function test
D
A 78-year-old woman visits her GP with a 4-month history of constipation and blood coating her stools. She has also lost 9 kg of weight and complains that she doesn’t ‘feel empty’ after defecating. Abdominal examination is normal, apart from an enlarged left supraclavicular lymph node. What is the most likely diagnosis? A Cancer of the rectum B Cancer of the sigmoid colon C Gastric carcinoma D Cancer of the caecum E Pancreatic cancer
A
A 75-year-old man is rushed into A&E by ambulance. He finds it difficult to answer simple questions and is struggling to speak. On examination, power is 2/5 in
his right arm, 4/5 in his right leg and 5/5 in his left arm and leg. He has marked facial muscle weakness on the right half of his face and he is blind in the right half of his visual field. A CT head scan is performed and an ischaemic stroke is diagnosed. Which artery is most likely to be involved?
A Right anterior cerebral artery
B Left anterior cerebral artery
C Right posterior cerebral artery
D Right middle cerebral artery
E Left middle cerebral artery
E
A 62-year-old diabetic on metformin sees his GP for a routine blood test. He claims that he has been compliant with his treatment and has not experienced any symptoms recently. His blood test reveals: Na+ : 116 mmol/L (135-145) K+ : 3.7 mmol/L (3.5-5) Ca2+ : 2.4 mmol/L (2.2-2.6) Total Cholesterol : 9.2 mmol/L (< 5) Serum Albumin : 48 g/L (35 -50) TFT - Normal SST - Normal What is the most likely cause of his hyponatraemia? A Addison's disease B Hypothyroidism C Erroneous result D Drug side-effect E Nephrotic syndrome
C
Which of the following lung pathologies produces the ‘sail sign’ appearance on CXR? A Right upper lobe collapse B Right middle lobe collapse C Right lower lobe collapse D Left upper lobe collapse E Left lower lobe collapse
E
A 47-year-old man has vomited 3 times and has not passed any faeces or flatus for the last 4 days. He had an open cholecystectomy 6 years ago but has otherwise been relatively fit and healthy. What is the best immediate management option for this patient? A NG tube and IV fluids B Surgery to resolve the obstruction C Gastrograffin D IV antibiotics E Reassure and discharge
A
Which of the following is not a major criterion in the Framingham criteria for congestive cardiac failure? A Bilateral ankle oedema B Paroxysmal nocturnal dyspnea C Cardiomegaly D S3 gallop E Acute pulmonary oedema
A
A 46-year-old female has experienced a painful sensation on the outer side of her left thigh for the past 3 months. She mentions that the sensation is very ‘bizarre’ and sometimes feels like it is burning or tingling. She has no other symptoms and has no past medical history of note. What is the most likely diagnosis? A Meralgia paraesthetica B Multiple sclerosis C Sciatica D Peripheral neuropathy E Disc herniation
A
A 47-year-old female suffering from RUQ pain, lethargy and pruritus, is found to have an ALP of 300 IU/L (30-150 IU/L) and serology is positive for anti-mitochondrial antibodies. She also complains of dry, itchy eyes. Examination findings include icterus and xanthelasma. What is the most likely diagnosis? A Type 1 autoimmune hepatitis B Type 2 autoimmune hepatitis C Primary sclerosing cholangitis D Primary biliary cirrhosis E Cirrhosis
D
A 53-year-old woman has been suffering from recurrent painful episodes affecting her face – mainly her right cheek. She describes the pain as being extremely intense, sharp and sudden, like an ‘electric shock’. It usually lasts for a few seconds before subsiding. The pain often occurs when she brushes her teeth. What is the most likely diagnosis? A Giant cell arteritis B Trigeminal neuralgia C Ramsay Hunt syndrome D Shingles E Cluster headache
B
A 21-year-old woman visits the GP complaining of a 2-month history of bloating and watery diarrhoea. She adds that she often has to rush to the toilet. During the consultation, she starts furiously itching her elbows. On examination, there is a blistering, papulovesicular rash covering both elbows. What is the most likely diagnosis? A Inflammatory bowel disease B Coeliac disease C Gastroenteritis D Irritable bowel syndrome E Ischaemic colitis
B
Which of the following is not a chest X-ray feature of heart failure? A Kerley B lines B Upper lobe diversion C Cardiomegaly D Pleural effusion E Air bronchograms
E
A 71-year-old man is referred to the oncology clinic having suffered from hip pain, constipation and abdominal pain for the past few months. He has also noticed that he is having to urinate more frequently than usual, and, consequently, he is always extremely thirsty. A full blood count and U&Es are requested:
Hb = 10.5 g/dL (13-18)
MCV = 106 fl (76-96)
Platelets = 120 x109/L (150-400 x 109)
Creatinine = 125 umol/L (baseline: 72 umol/L (3 months ago))
The oncologist requests a blood film. Considering the most likely diagnosis, what would you expect to see on this patient’s blood film?
A Rouleaux formation
B Schistocytes
C Granulocytes with absent granulation and hyposegmented nuclei
D Dacrocytes
E Smear cells
A
A 56-year-old woman, of Somalian origin, presents with a 2-month history of haemoptysis. She has also noticed some weight loss during this time and complains that she is having to change her bed sheets more often than usual as they are often drenched with sweat in the morning. Examination reveals painless cervical lymphadenopathy and tender, purple lumps on her shins. A CXR reveals an area of consolidation in the right upper lobe. Which investigation should be performed next in order to establish a diagnosis? A Bronchoalveolar lavage B Chest CT scan C Sputum sample for acid-fast bacilli D Bronchoscopy and biopsy E Mantoux test
C
. Which type of urinary tract stone is most common? A Magnesium ammonium phosphate B Calcium oxalate C Cysteine D Urate E Hydroxyapatite
B
A 62-year-old man presents with severe, acute epigastric pain with nausea and vomiting. The pain radiates to the back and improves when sitting forward. It started 4 days ago, but the patient assumed it was indigestion and refused to come to hospital. On examination, there is epigastric tenderness and ecchymoses over the periumbilical area and flank. The patient drinks in moderation and has not had any alcohol recently. Serum amylase is 600 U/L (< 140). Which investigation should be performed to confirm the diagnosis? A ERCP B Abdominal CT scan C Abdominal X-ray D Abdominal ultrasound E MRCP
D
A 39-year-old carpenter is brought into A&E having fallen from a 3rd floor balcony. He landed on his head and has been unconscious since the incident. An intracranial haemorrhage is suspected and an urgent CT scan is requested. The patient’s vital signs are recorded: BP = 195/120 mm Hg; HR = 47 bpm (60-100). His breathing also appears to be irregular – shallow breaths interspersed with periods of apnoea. What is the name given to this phenomenon? A Kussmaul sign B Cushing’s reflex C Beck’s triad D Charcot’s triad E Baroreceptor reflex
B
A 46-year-old woman visits A&E complaining of a fever and episodes of shivering. She returned from Nigeria 2 weeks ago and confesses that she was not very compliant with her antimalarial medication. Therefore, malaria is suspected. Which investigation should be performed to diagnose malaria?
A Thick and thin blood films
B Blood cultures
C Heterophile antibody test
D Enzyme-linked Immunosorbent Assay (ELISA)
E Urinalysis
A
A patient suffering an acute exacerbation of COPD has become hypoxic with an SaO2 of 83%. He requires administration of oxygen at a tightly regulated concentration. Which of the following methods of administering oxygen would be most appropriate? A Nasal cannula B Hudson face mask C Venturi mask D Non-rebreathing mask E BiPAP
C
A 48-year-old man has been suffering from frequent urination for the past 5 months. He has been going to the toilet around 10 times per day and he has been drinking excessive volumes of water. He has also been constipated for the past month with vague ‘tummy pains’ and complains of joint pain in his hands. A blood test is requested, which shows:
Na+: 137 mmol/L (135-145) K+: 4.6 mmol/L (3.5-5) Ca2+: 3.0 mmol/L (2.2-2.6) ALP: 197 iU/L (30-150) PTH: 102 ng/L (10-65)
What is the most likely diagnosis?
A Vitamin D Toxicosis B Parathyroid Adenoma C Paget’s Disease D Malignancy EMilk-Alkali Syndrome
B
A 61-year-old male comes to A&E complaining of chest pain and mentions that he can feel his heart ‘pumping out of his chest’. An ECG shows regular broad complex tachycardia, with no P waves. His blood pressure is 124/87 mm Hg. How should this patient be treated? A Defibrillation B DC cardioversion C Amiodarone D Adenosine E Atropine
C
A 30-year-old man presents to his GP complaining of a swollen scrotum, which he first noticed 3 weeks ago. He adds that the swelling feels like a ‘bag of worms’, and, despite being a little uncomfortable, it is not painful. On examination, the patient’s scrotum looks normal when he is supine, however, the left hemiscrotum becomes swollen when he stands up. The GP can get above the swelling and distinguish it from the testicle. What is the most likely diagnosis? A Indirect inguinal hernia B Direct inguinal hernia C Hydrocoele D Varicocoele E Epididymal cyst
D
A 74-year-old woman is brought to A&E having suffered several violent bouts of vomiting. On examination, she is clearly distressed and has a massively distended abdomen. When questioned, she struggles to answer but complains of generalised abdominal pain and mentions that she hasn't passed any faeces or flatus since the pain began. Bowel obstruction is suspected and an AXR is requested. The AXR shows a massively distended loop of large bowel which looks like an embryo. What is the most likely cause of this bowel obstruction? A Colorectal cancer B Sigmoid volvulus C Caecal volvulus D Adhesions E Femoral hernia
C
A 36-year-old female presents to the GP complaining that the nail bed of her ring finger has detached and she is worried that the same is happening to her other finger nails. The GP suspects onycholysis. Which of the following is not associated with onycholysis? A SLE B Psoriasis C Thyrotoxicosis D Trauma E Fungal infection
A
A 72-year-old man has recently suffered a stroke. He has recovered well and appears to have regained much of his physical strength, however, his speech has changed quite considerably. His daughter says that he will talk the same amount as he always did but his sentences will not make any sense, and he doesn’t seem to notice. When asked to describe what he did this morning, he responds: ‘the bugle fidget and that I played tractor to you before’. Damage to which part of the brain is likely to manifest in this way? A Wernicke’s area B Broca’s area C Arcuate fasciculus D Hippocampus E Amygdala
A
An inpatient on the surgical ward is recovering after having a kidney stone removed. A routine blood test is performed which shows the following results:
Na+ : 135 mmol/L (135 – 145)
K+ : 8.7 mmol/L (3.5 – 6.0)
Ca2+ : 0.2 mmol/L (2.2 – 2.6)
An ECG is performed which shows no obvious abnormalities.
Given above information, what should be the next step in the management of this patient?
A Urgently draw another blood sample
B 10 mL 10% calcium gluconate
C 20 mL 20% calcium gluconate
D 50 mL 50% dextrose + 10 U insulin
E IV salbutamol
A
What are the ‘B symptoms’ of lymphoma?
A Fever, Lymphadenopathy, Rigors
B Fever, Night Sweats, Weight Loss
C Recurrent Infections, Anaemia, Easy Bruising
D Night Sweats, Pruritus, Weight Loss
E Lymphadenopathy, Weight Loss, Loss of Appetite
B
A 40-year-old teacher has recently heard several distressing comments about how flushed she is looking. On examination, she has very red cheeks and a loud S1 with a mid-diastolic murmur is heard over the apex when the patient is in the left lateral position. What is the most likely diagnosis? A Mitral stenosis B Mitral regurgitation C Aortic stenosis D Aortic regurgitation E Tricuspid regurgitation
A
A 59-year-old female presents with epigastric pain that gets worse a few hours after eating. The patient has taken ibuprofen every day for the past 2 years for her chronic back pain. A urea breath test is negative. What is the most appropriate treatment option for this patient?
A Stop ibuprofen and give omeprazole
B Stop ibuprofen and give ranitidine
C Give amoxicillin, metronidazole and pantoprazole
D Give lifestyle advice and arrange to see the patient in 3 months
E Oral steroids
A
A 58-year-old woman presents to her GP with a 5-month history of worsening shortness of breath on exertion. She has also lost about 3 kg in weight and has experienced a dry cough. She has never smoked before and has a past medical history of rheumatoid arthritis, which was diagnosed 15 years ago. On examination, her fingers are clubbed and fine inspiratory crackles are heard bilaterally across the lower lung zones. What is the most likely diagnosis? A COPD B Lung cancer C Bronchiectasis D Pulmonary fibrosis E Tuberculosis
D
A 38-year-old woman has been experiencing palpitations, sweating and diarrhoea for the past week. Before these symptoms began, she was on sick leave for 3 days with a fever, sore throat and cough. During the consultation, she appears to be very anxious with a slight tremor in her hands. Vital signs: HR = 114, Temp =
38.6°C. A thyroid examination reveals a warm, tender and slightly enlarged thyroid gland. A thyroid uptake scan is requested. What would you expect to see?
A Diffuse uptake throughout an enlarged gland
B No uptake
C Multinodular gland with multiple hot nodules
D Multinodular gland with a single hot nodule
E Diffuse uptake with a single cold nodule
B
A 24-year-old waitress presents to her GP after noticing a small, firm lump in her left breast. She first noticed the lump 1 week ago and is unsure about whether it has grown. She reports no nipple discharge or skin changes over the breast. Examination reveals a 1 x 2 cm lump in the upper outer quadrant of the left breast with no axillary lymphadenopathy. She is worried because her grandmother died of breast cancer. What is the next most appropriate step in the management of this patient? A Urgent mammogram B Urgent ultrasound scan C Urgent CT Scan D Arrange elective mastectomy E Arrange elective wide local excision
B
A 73-year-old man was watching TV with his family, 2 hours ago, when his speech suddenly became slurred and he lost the ability to grip the remote control. His son, who accompanied him to A&E, noticed that the left half of his father’s face drooped during this episode. His symptoms eventually subsided after around 15 minutes and he claims that he feels back to normal, albeit slightly shaken by his ordeal. What is the first step in his management? A Administer aspirin B Thrombolysis C CT Head Scan D ECG E Carotid Endarterectomy
A
A 9-year-old girl is brought to see her GP, by her father, because she has been complaining of pain in her knees and ankles and a tummy ache, which began yesterday. Urinalysis is positive for blood and protein. On examination, small purple spots are seen on her buttocks and her knees feel warm and swollen. Her father
adds that, 2 weeks ago, she took time off school because of the flu. What is the most likely diagnosis?
A Post-infectious glomerulonephritis
B Immune thrombocytopenic purpura
C Disseminated intravascular coagulation
D Henoch- Schӧnlein purpura
E Minimal change glomerulonephritis
D
A 64-year-old woman presents with severe left iliac fossa pain with nausea and vomiting. She adds that she has had a few episodes of rectal bleeding recently where the blood has coated the stools. She admits to eating a lot of fast food and having a low-fibre diet. She has not noticed any weight loss. Her left iliac fossa is tender on palpation and blood is found on DRE. What is the most likely diagnosis? A Gastroenteritis B Diverticulitis C Angiodysplasia D Colorectal carcinoma E Inflammatory bowel disease
B
Which of the following organisms is responsible for causing whooping cough? A Bordatella pertussis B Treponema pallidum C Cryptosporidium D Mycoplasma pneumoniae E Yersinia pestis
A
A 59-year-old man is brought to A&E by his daughter after having collapsed at home. He has very little recollection of the incident – one minute he was doing the dishes, and next minute he was lying on his back on the floor. He has no history of recent head trauma and mentions that he felt ‘absolutely fine’ when he regained consciousness. An ECG is performed showing bradycardia (36 bpm) and dissociation between the p waves and QRS complexes. A diagnosis of complete heart block is made. What might be seen on close inspection of his JVP? A Large V waves B Cannon A waves C Kussmaul sign D Raised JVP with absent pulsation E Slow Y descent
B
A 36-year-old supermarket manager has been suffering from chronic back pain and stiffness. He first saw his GP about this matter 6 months ago and was given paracetamol, however, the pain worsened and has started affecting his job. He finds restocking the shelves particularly difficult as it requires repetitively bending down and picking up products. He mentions that the pain and stiffness is worst in the morning and gradually gets better with activity. What is the most likely diagnosis? A Ankylosing spondylitis B Lumbar disc herniation C Osteoarthritis D Muscle strain E Vertebral fracture
A
A 69-year-old man is recovering in the inpatient respiratory ward having been diagnosed with pneumonia yesterday. A right-sided pleural effusion is identified on the chest X-ray. Which of the following findings on clinical examination of the right lung base, would be most consistent with a right-sided pleural effusion?
A Resonant percussion note, increased vocal resonance, vesicular breathing
B Resonant percussion note, reduced vocal resonance, reduced breath
sounds
C Dull percussion note, increased vocal resonance, bronchial breathing
D Dull percussion note, decreased vocal resonance, reduced breath sounds
E Dull percussion note, decreased vocal resonance, vesicular breathing
D
A 34-year-old woman is complaining of a drooping eyelid that has been affecting her vision. She has also been suffering from fatigue over the past 3 months, which has impacted on her job as a yoga instructor. She feels relatively fine in the morning, however, she feels very weak towards the end of the day and struggles to complete her evening sessions. What is the most likely diagnosis? A Polymyalgia rheumatic B Anaemia C Myasthenia Gravis D Lambert-Eaton syndrome E Horner’s syndrome
C
A 19-year-old asthmatic visits his GP because he is having to use his salbutamol inhaler more than 3 times per week. What is the next step in the management of this patient’s asthma?
A Add oral prednisolone
B Reassure and send home
C Increase the dose of inhaled salbutamol
D Add inhaled salmeterol
E Add inhaled low-dose beclomethasone
E
A 71-year-old owner of a dye factory presents to his GP having experienced 3 episodes of blood in his urine over the past week. When asked to elaborate, he says that his urine is bright red, however, he experiences no pain when passing urine and has not experienced any trauma to his genitals recently. Otherwise, he has generally been quite healthy although he has noticed that his clothes have become quite loose-fitting despite not having changed his diet or exercised. What is the most likely diagnosis?
A Pyelonephritis B Glomerulonephritis C Bladder Cancer D Prostate Cancer E Ureteric Stone
C
A 44-year-old woman presents with a 7-month history of heartburn, an acidic taste in the back of her mouth and painful swallowing. The GP suspects gastro- oesophageal reflux that is aggravated by a medication that she is taking for a heart CONDITION. Which of the following options could cause or worsen gastro-oesophageal A Beta-blockers B ACE inhibitors C Nitrates D Diuretics E Angiotensin receptor blockers
C
A 27-year-old man presents with palpitations and light-headedness. An ECG shows features consistent with a supraventricular tachycardia. Adenosine is administered and the SVT is terminated. A repeat ECG shows a short PR interval and a QRS complex with a slurred upstroke. What is the diagnosis? A Brugada syndrome B LBBB C Romano-Ward syndrome D Wolff-Parkinson-White syndrome E Complete heart block
D
An 18-year-old man visits his GP complaining of an itchy scalp and nose. He admits that he has been feeling quite self-conscious since a friend pointed out that he has dandruff. On examination, there are patchy erythematous plaques along his scalp covered with yellow scales and white flakes of dead skin in his hair. Similar lesions are also found in the nasolabial folds. Which type of eczema is this likely to be? A Nummular B Seborrhoeic C Contact D Atopic E Pompholyx
B
Which of the following options fits the criteria for giving long-term oxygen therapy in COPD?
A PaO2 7.3-10 kPa despite maximal treatment
B PaO2 7.3-10 kPa and pulmonary hypertension
C PaO2 < 7.3 kPa despite maximal treatment
D PaCO2 > 6 kPa despite maximal treatment
E PaCO2 > 8 kPa despite maximal treatmen
C
A 71-year-old female, with a history of atrial fibrillation, presents to A&E with severe, diffuse abdominal pain. Her blood pressure is 84/60 mm Hg and her pulse is irregularly irregular with a rate of 130 bpm. Abdominal examination is normal. An abdominal X-ray is performed. Which of these radiological features is most likely to be seen? A Rigler’s sign B Pneumoperitoneum C Toxic megacolon D Gasless abdomen E Coffee bean sign
D
Which of the following results would you expect to see in a patient with toxic multinodular goitre? A High TSH, High TRH & High T3/T4 B Low TSH, Low TRH & High T3/T4 C Low TSH, High TRH & High T3/T4 D High TSH, Low TRH & High T3/T4 E High TSH, High TRH & Low T3/T4
B
A 68-year-old man has suddenly developed an extremely painful left leg. On examination, his left leg is pale, cold and his dorsalis pedis and posterior tibial pulses are impalpable. His radial pulse is 120 bpm and has an irregularly irregular rhythm. What is the first step in the management of this patient?
A Duplex ultrasound scan of the lower limb vessels
B Oral aspirin
C IV heparin
D Measure Ankle-Brachial Pressure Index (ABPI)
E DC Cardioversion
C
A 22-year-old man presents with a headache, neck stiffness and photophobia. A diagnosis of viral meningitis is suspected. Once raised ICP is excluded, a lumbar puncture is performed. Which set of results would be consistent with viral meningitis?
A Cloudy CSF, high neutrophils, high protein and low glucose
B High lymphocytes, high protein and normal glucose
C High lymphocytes, low protein and normal glucose
D High neutrophils, high protein and high glucose.
E Fibrinous CSF, high lymphocytes, high protein and low glucose
B
Which of the following is not a clinical feature of anaemia? A Conjunctival pallor B Glossitis C Angular stomatitis D Ruddy/Red complexion E Shortness of breath
D
A 39-year-old female presents at her GP practice having coughed up blood last night. This has happened on two previous occasions. She has no significant past medical history although she does experience regular nosebleeds. Blood tests reveal a high ESR and urinalysis reveals proteinuria and haematuria. The presence of which antibody would support the most likely diagnosis?
A Anti-GBM antibody
B pANCA
C cANCA
D Anti-liver/kidney microsomal antibody
E Anti-smooth muscle antibody
C
Which of the following is not part of the criteria for diagnosing sepsis?
A Heart Rate > 90 bpm
B Respiratory Rate > 20 breaths per minute
C Temperature > 38°C
D White Cell Count < 4 x 109/L
E Blood Pressure < 90/60 mm Hg
E
A 72-year-old patient with severe COPD has recently experienced worsening dyspnoea despite maximal treatment. On examination, he is cyanotic with a raised JVP and ankle oedema. Palpation reveals hepatomegaly. What is the most likely diagnosis? A Left ventricular failure B Congestive cardiac failure C Cor pulmonale D Pulmonary hypertension E Restrictive cardiomyopathy
C
A 32-year-old man with psoriasis presents to his GP with deformed hands that have been affecting his ability to do his daily tasks. It has gradually got worse over several years. On closer inspection, his fingers are badly deformed and appear to be telescoped. What is the most likely diagnosis? A Rheumatoid arthritis B Arthritis mutilans C Psoriatic spondylopathy D Osteoarthritis E Distal interphalangeal joint disease
B
A 21-year-old woman has fainted 4 times in the past 3 months. She becomes sweaty and nauseous before she faints and is usually unconscious for a few seconds. Her friends have told her that she looks abnormally pale before she collapses. She doesn’t know if she jerks whilst unconscious, but has not lost control of her bladder or bitten her tongue. When she regains consciousness, she feels slightly dizzy but does not feel confused. What is the most likely cause of her fainting?
A Hypoglycaemia
B Epileptic seizure
C Vasovagal syncope
D Arrhythmia
E Hypertrophic obstructive cardiomyopathy
C
A 50-year-old taxi driver attends a GP appointment because he has recently been ‘bumping into things’ quite regularly and has had to take a break from work over fears about his vision. He struggles to see anything in the left half of his visual field. Examination reveals a left homonymous hemianopia. In which part of the visual pathway is the lesion likely to be located? A Optic chiasm B Left optic tract C Left optic radiation D Right optic nerve E Right optic tract
E
. A 75-year-old woman has had a 3-week history of lower abdominal discomfort and bloating. She is embarrassed to admit that she has recently started wearing adult diapers because she has been soiling her underwear. Her stools are usually very watery and drip into the pan. She has not lost any weight or noticed any blood in the stool. She claims to have a balanced, healthy diet. She has taken codeine every day for the past 4 months since she suffered a hip fracture. On examination, her abdomen is mildly distended and a solid mass is palpated in the left iliac fossa. On digital rectal examination, her underwear is soiled and liquid stool is seen on withdrawal of the finger. What is the most likely diagnosis? A Rectal carcinoma B Faecal impaction C Inguinal hernia D Ischaemic colitis E Rectocoele
B
- A 24-year-old female presents to her GP complaining that her periods have become extremely irregular. She normally has 26-29 day cycles, but in the past 6 months her periods have been much less frequent. On closer inspection, she appears to have an abnormally large amount of facial hair for a young female and she is also suffering from acne, which, she claims, she never had as a teenager. She has gained weight over the past few months, which, alongside the acne and facial
hair growth, has made her feel depressed. What is the most likely diagnosis?
A Hypothyroidism
B Turner’s syndrome
C Polycystic ovarian syndrome
D Pregnancy
E Panhypopituitarism
C
Which of the following is a respiratory cause of asterixis? A Hypoxia B Carbon dioxide retention C Salbutamol side-effect D Secondary polycythaemia E Bronchiectasis
B
A 23-year-old student has arranged an appointment with his GP to discuss his ‘incredibly itchy’ eyes. Yesterday morning he noticed that his left eye became quite red and itchy, and started watering. A few hours later, his right eye also started to display the same symptoms. On closer inspection, both eyes show conjunctival injection and watering. A yellow crust is seen across the margins of the eyelids. What is the most likely diagnosis? A Hypopyon B Hyphaema C Bacterial Conjunctivitis D Viral Conjunctivitis E Uveitis
C
A 53-year-old man visits his GP to discuss a swollen scrotum that has caused him some discomfort and much embarrassment since he first noticed it 3 weeks ago. The swelling has grown gradually and, although it is uncomfortable, it is not painful. He reports no difficulties with passing urine. On examination, his left hemiscrotum is considerably enlarged, fluctuant and non-tender. It is possible to get above the swelling, however, the left testicle cannot be distinguished. When a pen torch is shone on the swelling, it illuminates brightly. What is the most likely diagnosis? A Varicocoele B Hydrocoele C Testicular tumour D Epididymal cyst E Indirect inguinal hernia
B
- A 52-year-old man has been experiencing some chest pain and shortness of
breath, which is worse when lying down. He has also collapsed 3 times in the past couple of months. His father died of a heart condition when he was 58 years old, although he cannot recall the details of the condition. On examination, a jerky carotid
pulse is palpated and a crescendo-decrescendo murmur is heard over the carotid artery. What is the most likely diagnosis?
A Aortic stenosis
B Hypertrophic obstructive cardiomyopathy
C Left heart failure
D Mitral regurgitation
E Constrictive pericarditis
B
A 48-year-old man has been in hospital for 2 days receiving treatment for pneumonia. He has a past medical history of acute pancreatitis, which occurred 3 years ago. He has a long history of alcohol abuse. In the last hour, he has started sweating excessively, complains of palpitations and appears very anxious. He is clearly agitated and begins shouting at the nurses to ‘get these creatures off me!’ What is the most appropriate treatment? A Diazepam B Phenobarbital C Loperamide D Chlordiazepoxide E Risperidone
D
A 56-year-old man has recently registered at a new GP practice. As part of the registration process he has been asked to undergo some routine blood tests. FBC reveals the following results:
Hb : 106 g/L (130-180)
WBC : 95 x 109 /L (4-11) Platelets : 86 x 109 /L (150-400) Lymphocytes : 85 x 109 (1.5-4.5)
A diagnosis of chronic lymphocytic leukaemia is suspected. Which of the following features are you most likely to see on his blood film?
A Smear cells
B Atypical lymphocytes
C Auer rods
D Reed-Sternberg cells
E Schistocytes
A
A 21-year-old university student presents with a 1-week history of sore throat, fever and malaise. On examination, there is cervical lymphadenopathy, splenomegaly and inflamed tonsils. The GP diagnoses the patient with bacterial tonsillitis and prescribes ampicillin. The patient comes back 2-days later with a widespread maculopapular rash. What is the underlying diagnosis? A Penicillin allergy B Erythema multiforme C Stevens-Johnson syndrome D Infectious mononucleosis E Idiopathic thrombocytopaenic purpura
D
A 47-year-old man presents to his GP having experienced a few episodes of haemoptysis over the past month. He returned from a holiday to Bangladesh 6 weeks ago. On direct questioning, he admits to losing approximately 5 kg in weight over the past month and he has had to replace his bed sheets more frequently because they are often drenched with sweat when he wakes up in the morning. A chest X-ray reveals an area of consolidation in the right upper zone. Sputum microscopy using Ziehl-Neelsen stain reveals acid-fast bacilli. What is the most appropriate treatment option?
A Rifampicin and isoniazid for 6 months; ethambutol and pyrazinamide for the first 2 months
B Ethambutol and pyrazinamide for 6 months; rifampicin and isoniazid for the first 2 months
C Rifampicin and isoniazid for 6 months; ethambutol and pyrazinamide for the first 4 months
D Rifampicin and pyrazinamide for 6 months; ethambutol and isoniazid for the first 2 months
E Rifampicin, pyrazinamide, ethambutol and isoniazid for 6 months
A
- A 23-year-old university student is brought to A&E at 2 am by his friends. He is
clearly inebriated and struggles to maintain conversation. His friends explain that he had been celebrating the recent election results at a bar when he began to vomit. He vomited several times and splashes of ‘bright red’ blood was seen the last two times that he vomited. What is the most likely diagnosis?
A Peptic ulcer disease
B Boerhaave syndrome
C Mallory-Weiss syndrome
D Gastritis
E Osler-Weber-Rendu Syndrome
C
A 65-year-old man is brought in to A&E by his wife. She says that he has been very confused over the last few days and has fallen over several times. She adds that her husband has wet the bed twice over the last 2 days – this has never happened before. What is the most likely diagnosis? A Alzheimer’s disease B Obstructive hydrocephalus C UTI D Subdural haematoma E Normal pressure hydrocephalus
E
Which of the following is a feature of limited cutaneous systemic sclerosis? A Calcinosis B Cyanosis C Striae D Onycholysis E Clubbing
A
A 32-year-old basketball player is brought to A&E extremely breathless. He was at basketball training when he suddenly felt himself getting more and more breathless and developed a ‘stabbing’ pain on the right side of his chest. He has never experienced anything like this before. On examination, he is very tall and thin, and breath sounds are reduced over the right side of his chest. What is the most likely diagnosis? A PE B Primary pneumothorax C Secondary pneumothorax D Myocardial infarction E Asthma attack
B
A 22-year-old student presents with a severe headache and fever that has lasted 1 day. On examination, he has a stiff neck and a rash across his arms and legs. The junior doctor gently flexes the patient’s neck. As he does this, the patient’s hips flex. What is the name of this sign? A Uhthoff’s sign B Lhermitte’s sign C Kernig’s sign D Brudzinski’s sign E Tinel’s sign
D
A 61-year-old woman visits the GP complaining of 13 kg of weight loss over the past 6 months. On direct questioning, she admits that her faeces are lighter in colour than normal and her urine has become darker. She is jaundiced and a large non- tender mass is palpated in her right upper quadrant. What is the most likely diagnosis? A Gallstones B Hepatocellular carcinoma C Pancreatic cancer D Bile duct stricture E Cirrhosis
C
A 46-year-old man has been admitted to A&E after experiencing palpitations, which began about 4 hours ago. An ECG is performed, which reveals atrial fibrillation. He has no previous history of ischaemic heart disease. He refuses DC cardioversion. What is the next most appropriate treatment option? A Defibrillation B Low molecular weight heparin C Warfarin D Flecainide E Digoxin
D
A 4-year-old boy is referred to the paediatric department by his GP after a 3-week history of fatigue, shortness of breath and recurrent chest infections. A thorough examination is performed, which revealed extensive bruising across the child’s body, hepatosplenomegaly and cervical lymphadenopathy. Based on the information provided, what is the most likely diagnosis? A Acute lymphoblastic leukaemia B Acute myeloid leukaemia C Chronic lymphocytic leukaemia D Chronic lymphoblastic leukaemia E Hodgkin’s lymphoma
A
A 79-year-old woman is accompanied by her granddaughter to A&E. She has had a productive cough and a fever for the past 4 days. On examination, she has an AMTS of 5/10, respiratory rate of 31/min and blood pressure of 92/66 mm Hg. Her urea is 3 mmol/L (2.5-6.7). A CXR reveals an area of consolidation in the right middle lobe. Community-acquired pneumonia is suspected. What is her CURB-65 score?
A There is not enough information to tell.
B2
C3
D4
E5
C
A 28-year-old man has experienced several episodes of sweating, palpitations and anxiety over the past 6 months. He has a past medical history of thyroid cancer (aged 19) which was treated with total thyroidectomy. What is the most appropriate investigation?
A Serum 17-hydroxyprogesterone levels
B 24 hr urine 5-hydroxyindoleacetic acid levels
C 24 hr urine vanillylmendelic acid
D Plasma adrenaline concentration
E Thyroid uptake scan
C
Which of the following triads best describes Horner’s syndrome? A Ptosis, miosis, anhydrosis B Proptosis, miosis, hyperhidrosis C Ptosis, mydriasis, anhydrosis D Ptosis, enophthalmos, hyperhidrosis E Proptosis, miosis, anhydrosis
A
Which virus is implicated in around 50% of cases of Hodgkin’s lymphoma? A Human cytomegalovirus B Herpes simplex virus 2 C Varicella zoster D Epstein-Barr virus E Human herpesvirus 7
D
A 54-year-old man is complaining of abdominal heaviness and shortness of breath. On examination, his abdomen is distended, non-tender and exhibits shifting dullness with a fluid thrill. The junior doctor suspects ascites and requests a diagnostic paracentesis. It reveals a Serum-Ascites Albumin Gradient (SAAG) of 9 g/L. Which of the following is a potential cause of his ascites? A Cirrhosis B Congestive cardiac failure C Portal hypertension D Budd-Chiari syndrome E Nephrotic syndrome
E
A 56-year-old man has been waking up several times at night to empty his bladder. He says he doesn’t feel completely empty after finishing and his stream seems to be quite ‘stop and start’. He often has to strain to maintain the flow. Which of his symptoms is considered irritative? A Incomplete emptying B Having to start and stop C Increased urination at night D Straining E Weak flow
C
An 8-year-old boy is brought to the GP by his mother. He has a very swollen and painful knee which arose yesterday without any preceding trauma. On closer inspection, he is afebrile and the joint, despite being swollen, does not appear inflamed. He also has several bruises across his torso. His mother mentions that her father suffered from haemophilia and that she is worried that her son may have the same disease. Blood tests are requested. Which of the following results would be suggestive of a diagnosis of haemophilia? A High APTT, Normal PT B Normal APTT, High PT C High APTT, High PT D Low bleeding time E Low vWF
A
An 82-year-old man is brought into A&E complaining of severe flank pain that started suddenly about 30 minutes ago. On examination, he looks very unwell and his palms are cold and sweaty. Vital Signs: HR = 132 bpm; BP = 84/52 mm Hg. What is your top differential? A Myocardial infarction B Ruptured abdominal aortic aneurysm C Ureteric colic D Disc prolapse E Muscle sprain
B
A 31-year-old lady, who is 7 months pregnant, is brought to A&E having become extremely short of breath this morning. She has also experienced sharp chest pain on her right side. Examination reveals no abnormalities and an ECG shows sinus tachycardia. A pulmonary embolism is suspected. What is the most appropriate investigation to request? A D-Dimer B CTPA C VQ scan D Chest X-ray E Doppler ultrasound of the lower limbs
C
A 76-year-old woman is brought to A&E by her daughter. She is complaining of severe left iliac fossa pain accompanied by nausea, vomiting and fever. On inspection, she shows signs of peritonism. Vital signs: HR = 123 bpm, RR = 24 bpm, Temp = 38.7°C and BP = 87/54 mm Hg. An erect CXR is requested, which shows air under the diaphragm. A diagnosis of perforated diverticulitis localised to the sigmoid colon is made. What is the most appropriate surgical procedure? A Left colectomy B Abdominoperineal resection C Hartmann’s procedure D Delorme procedure E Anterior resection
C
Which of the following tumour markers is associated with ovarian cancer? A CA 15-3 B CA 19-9 C CA125 D CEA E aFP
C
A 61-year-old man visits his GP complaining of a ‘shooting pain’ in his legs. The pain comes on when he walks his dog, and it gets particularly bad when walking downhill. On questioning, he reveals that he has been urinating about 10 times every day, which is much more than usual. On examination, there is a loss of sensation up to the T10 vertebral level, increased tone in his legs and brisk ankle jerks. The GP also notices that the patient has a stooped posture. What is the most likely diagnosis? A Benign prostate hypertrophy B Motor neuron disease C Sciatica D Spinal cord stenosis E Cauda equina syndrome
D
An 11-year-old girl, who has recently moved to the UK from Cambodia, is referred to the cardiology department after her GP identified a heart murmur a few weeks after diagnosing her with a throat infection. She has also experienced intermittent joint pain, mainly affecting her knees and hips. On examination, she has a mid-diastolic murmur heard loudest over the mitral area and a few small, mobile nodules are palpated along the ulnar border of her forearms. What is the most likely diagnosis? A Infective endocarditis B Rheumatic fever C Septic arthritis D Rheumatoid arthritis E Lyme disease
B
A 12-year-old boy is brought into A&E – he is extremely drowsy, appears dehydrated and has vomited whilst in the ambulance. He is also clutching his abdomen and appears to be in considerable pain. He is a known diabetic, and DKA is suspected. The patient begins breathing in a very deep and laboured manner. What is the name given to this pattern of breathing? A Cheyne-Stokes breathing B Hypoventilation C Kussmaul breathing D Biot’s respiration E Apnoea
C
A 21-year-old man has been experiencing some scrotal discomfort over the past month, which he describes as feeling ‘heavier than usual’. On examination, a firm, non-tender lump can be palpated at the base of the right testicle. The patient had an undescended testicle as a child, which was corrected with orchidopexy. A diagnosis of testicular cancer is suspected. The registrar recommends performing a CT scan to assess for spread. Which group of lymph nodes does testicular cancer spread to? A Inguinal B Femoral C Para-aortic D Iliac E Mesenteric
C
Which of the following matches the criteria for type 2 respiratory failure? A PaO2 < 10.5 kPa, PaCO2 > 6 kPa B PaO2 <8kPa,PaCO2 >6kPa C PaO2 < 10.5 kPa, PaCO2 > 8 kPa D SaO2 < 90%, PaCO2 < 8 kPa E SaO2 <90%,PaO2 <8kPa
B
A 26-year-old model comes to see her GP after having noticed some blood streaked on the paper after emptying her bowels. This started two weeks ago. She adds that defecation is very painful. When asked about her diet, she reveals that she often eats ready meals and drinks relatively little water because her job involves regular travelling making it difficult for her to maintain a healthy diet. What is the most likely diagnosis? A Haemorrhoids B Anal fissure C Anal fistula D Colorectal cancer E Ulcerative colitis
B
A 71-year-old man presents with an 8-month history of worsening shortness of breath on exertion, orthopnoea and a cough productive of pink, frothy sputum. He has a past medical history of ischaemic heart disease and type 2 diabetes mellitus. Heart failure is suspected. What is the best investigation to confirm a diagnosis of heart failure? A ECG B Brain natriuretic peptide C Atrial natriuretic peptide D Echocardiogram E Chest X-ray
D
An inpatient on the respiratory ward is currently undergoing treatment for a pneumonia that he developed 2 days ago. A blood test is performed which shows a low white cell count, with a particularly low neutrophil count. The patient is re- examined and found to have a considerably enlarged spleen. On further questioning, He has suffered from 3 infections in the past 5 months and complains that his rheumatoid arthritis has been getting worse. What is the most likely diagnosis? A Malaria B Tuberculosis C Felty’s syndrome D Lymphoma E Chronic lymphocytic leukaemia
C
Which stain is used when testing for TB? A Giemsa B Gram C India Ink D Sudan Black E Ziehl-Neelsen
E
A 60-year-old man, with a history of hypertension and type 1 diabetes mellitus, is brought to A&E by his daughter. She says that 3 hours ago, when they were eating dinner, he suddenly dropped his fork and started slurring his words. On examination, the right side of his face is drooping, muscle power is 1/5 in the right arm and 5/5 in the left; 3/5 in the right leg and 5/5 in the left. What is the most appropriate management option?
A CT head to exclude bleed, then treatment dose of warfarin
B CT head to exclude bleed, then give antiplatelets and perform a swallow
assessment
C CT head to exclude bleed, then IV thrombolysis
D Control blood pressure and IV mannitol
E Craniotomy and evacuation
A
A 42-year-old wildlife photographer returns from a 6-month trip to South Africa. He has noticed a small, dark mole on his right calf, which, he claims, has not always been there. Examination reveals an asymmetrical, dark lesion with irregular borders that measures 1 cm in diameter. Malignant melanoma is suspected and an excisional biopsy is taken and sent to the pathologist. Which feature of the histological analysis is the most useful prognostic indicator in this situation? A Number of mitoses B Surface area of lesion C Depth of lesion D Mass of lesion E Lymphocyte infiltration
C
An 85-year-old man is brought to A&E having been found on a roundabout in the middle of the night. He is very confused with an AMTS of 4/10. U&Es are requested, which reveal hyponatraemia (Na+ : 118 mmol/L (135-145)). Care is taken to increase the sodium concentration slowly. What is a major consequence of raising plasma sodium concentration too rapidly? A Stroke B Rhabdomyolysis C Central pontine myelinolysis D AKI E Cerebral oedema
C
Which of the following antibodies is most sensitive for primary sclerosing cholangitis? A AMA B ASLA C ALKM-1 D pANCA E ANA
D
Which of the following is not a sign of cirrhosis? A Gynaecomastia B Asterixis C Koilonychia D Hepatic fetor E Clubbing
C
A 67-year-old pensioner, with a 40 pack-year smoking history, visits his GP complaining of shortness of breath that has gradually been getting worse over the past 6 months. He used to be able to walk 500 metres to the shops but now he struggles to make it up the stairs at home. He has also been suffering from a persistent cough productive of clear sputum. Which investigation is required to confirm the diagnosis? A Spirometry B Peak Expiratory Flow C Sputum Culture D Chest X-Ray E Bronchoscopy and Biopsy
A
An 18-year-old man visits the GP having recently returned from a holiday to Thailand. He complains of a 2-day history of watery, bloody diarrhoea and has vomited 6 times. On examination, he is pyrexial with diffuse abdominal tenderness. Which organism is most likely to be causing his symptoms? A Giardia lamblia B Vibrio cholera C Norovirus D E. coli 0157 E Salmonella
D
A 42-year-old man is brought into A&E by his wife. He is clutching his head and appears to be drowsy and distressed. Though a clear history is difficult to ascertain, he mentions that he has an ‘absolutely devastating headache’ that suddenly came on this morning. He has never experienced pain like this before. Towards the end of
the consultation he begins to vomit. He is apyrexial and denies any trauma to the head. On examination, the patient’s neck is slightly stiff, he is hypertensive and has large bilateral palpable masses in his abdomen. On direct questioning, he reveals that his father died suddenly at the age of 49. Which underlying disease has predisposed the patient to this clinical scenario?
A Renal cell carcinoma
B Polycystic kidney disease
C Medullary sponge kidney
D Phaeochromocytoma
E Subdural haemorrhage
B
A 33-year-old man has recently been diagnosed with hypertension following the incidental finding of abnormally high blood pressure during a routine check-up at his GP practice. Without further investigation, he was started on Ramipril. 1 week later, he begins to feel very nauseous and vomits several times. He is taken to A&E where his renal function is monitored:
Urea : 8.1 mmol/L (2.5-6.7)
Creatinine : 240 micromol/L (baseline : 102) eGFR : 53 (> 90)
Urine Output : 20 mL/hour (> 0.5 mL/kg/hr) What is the most likely cause of his condition?
A Acute tubular necrosis
B Acute interstitial nephritis
C Glomerulonephritis
D Renal artery stenosis
E Vasculitis
D
A 47-year-old man comes to A&E having experienced palpitations. He has a past medical history of hypertension which is being treated with ramipril and spironolactone. An ECG shows tented T waves and flattened P waves. A blood test reveals:
Na+ : 137 mmol/L (135-145) K+ : 6.8 mmol/L (3.5-5) Ca2+ : 2.3 mmol/L (2.2-2.6) pH : 7.35 (7.35-7.45)
What is the first step in the management of this patient?
A IV salbutamol
B 50 ml 50% dextrose with 10U insulin
C 50 ml 5% dextrose with 10U insulin
D 10 ml 10% calcium gluconate
E IV sodium bicarbonate
D
- Whilst eating dinner with his family, an 11-year-old boy suddenly drops his cutlery
and begins to stare blankly into space. His eyelids begin to flutter, his eyes roll upwards and this continues for 10 seconds. His dad notices the event and asks him about it, but he can’t remember what happened. What type of seizure is this describing?
A Absence
B Simple partial
C Complex partial
D Myoclonic
E Atonic
A
A 71-year-old man has had a 2-week history of shortness of breath that improves when lying flat. On examination, the patient has an oxygen saturation of 88%, his fingers are clubbed and there are multiple spider naevi on his chest. Shifting dullness is demonstrated, his spleen is enlarged and there are dilated veins around his umbilicus. The patient has a history of alcohol abuse. What is the most likely diagnosis? A Congestive cardiac failure B Portal hypertension C Hepatopulmonary syndrome D GI bleed E Alcoholic hepatitis
C
- A 72-year-old man is admitted to the orthopaedic surgery ward after fracturing his distal humerus whilst gardening. He mentioned that he suddenly felt a severe pain in his right arm and denies any significant preceding trauma. He adds that he has
recently been urinating a lot more frequently (often up to 12 times per day) and has suffered from constipation, which he attributes to being ‘part of growing old’. An X- ray of the affected arm reveals a pathological fracture with lytic deposits throughout the bone.
Blood tests reveal:
ESR = 48 mm/hr (0-22) Ca2+ = 3.1 mmol/L (2.2-2.6)
What is the most likely diagnosis?
A Multiple myeloma
B Paget’s disease
C Osteoporosis
D Vitamin D deficiency
E Thyrotoxicosis
B
A 49-year-old man is referred to the respiratory department by his GP. He has been suffering from gradual-onset, worsening shortness of breath over the past 3 months. His 50 pack-year smoking history makes COPD the top differential. Spirometry confirms these suspicions. What is the most appropriate first step in the pharmacological management of this patient? A Inhaled corticosteroid B Inhaled ipratropium bromide C Inhaled tiotropium D Symbicort E Long-term oxygen therapy
A
Which of the following is a sign of a lower motor neuron lesion? A Hyperreflexia B Spasticity C Fasciculations D Clonus E Babinski’s sign
C
A 47-year-old man has been suffering from rhinitis and recurrent nosebleeds for the past 3 months. At first, he attributed this to the cold weather, however, over the last 3 weeks he has started coughing up a small amount of blood. A series of bedside tests are performed, including a urine dipstick, which reveals proteinuria and haematuria. Blood tests and antibody screens reveal a raised ESR and cANCA. What is the most likely diagnosis? A Microscopic polyangiitis B Goodpasture’s syndrome C Granulomatosis with polyangiitis D Churg-Strauss syndrome E Behçet’s disease
C
Which of the following organisms most commonly causes gas gangrene? A Streptococcus pyogenes B Staphylococcus aureus C Staphylococcus epidermidis D Clostridium perfringens E Haemophilus influenzae
D
A 16-year-old schoolgirl books an appointment with her GP after noticing a lump in her left breast during self-examination. She, sensibly, decided to seek medical attention. She complains of no other symptoms. On examination, a 1 cm x 1 cm firm, smooth and very mobile lump is palpated in the upper-outer quadrant of her left breast. There is no pain on palpation nor is there any axillary or cervical lymphadenopathy. What is the most likely diagnosis? A Fibrocystic disease B Fibroadenoma C Breast cancer D Breast abscess E Fat necrosis
B
Which of the following drug classes is most likely to cause iatrogenic hypoglycaemia in diabetes patients? A Sulfonylureas B Metformin C Glucagon D Hydrocortisone E Orlistat
A
- A 24-year-old swimwear model presents to A&E with severe right iliac fossa pain. The pain was initially poorly localised to the umbilical region, before moving to
the right iliac fossa. This has been accompanied by nausea, anorexia and fever. A diagnosis of appendicitis is made and she is referred for an appendicectomy. What is the most suitable surgical incision for this patient?
A Lanz
B Kocher
C Pfannenstiel
D Rutherford-Morrison E Gridiron
A
- A 55-year-old man presents to his GP having coughed up blood on several
occasions over the past 6 months. He said that he has had a ‘smokers’ cough’ for years but the appearance of blood has been a recent change. On direct questioning, he admits to unintentionally losing about 5 kg of weight over the past 6 months. A chest X-ray shows a 2 cm cavitating lesion in the right upper lobe. There appears to be a few other smaller nodules surrounding the large cavitating lesion. The left lung appears slightly fibrosed, but is otherwise normal. What is the most likely diagnosis?
A Small cell lung cancer
B Squamous cell lung cancer
C Atypical pneumonia
D Lung abscess
E Goodpasture’s syndrome
B
- A 46-year-old airline pilot presents to A&E with severe pain in his right flank. He
adds that the pain moves down towards his right groin. Though examination is difficult, as he is writhing around in pain, no abnormalities are detected. However, a urine dipstick reveals haematuria. Which investigation would you do next?
A Renal ultrasound
B Cystoscopy
C CT-KUB
D MRI
E Urine MC&S
C
A 32-year-old man, with a history of IV drug abuse, presents to A&E with a high fever and rigors. He has also been very breathless and has experienced epigastric pain that is worse on exertion. On examination, giant V waves are seen in the JVP and tender, pulsatile hepatomegaly is palpated. What is the most likely diagnosis? A Mitral stenosis B Tricuspid regurgitation C Pulmonary hypertension D Portal hypertension E Viral hepatitis
B
A 31-year-old female presents to her GP with a 2-month history of fatigue and worsening muscle weakness. She also complains that her left eyelid droops considerably more than her right. She feels fine in the morning but her strength decreases throughout the day, especially if she exerts herself more so than usual. Myasthenia gravis is suspected. Which of the following investigations may provide evidence supporting this diagnosis? A Dix-Hallpike test B Schirmer’s test C Romberg’s test D Tensilon test E Trendelenberg test
D
A 15-year-old school boy is rushed into A&E having accidentally ingested some peanuts, to which he is extremely allergic. His face and lips are swollen, he is wheezing and struggling to breathe. Vital signs: HR = 132 bpm, BP = 88/53 mm Hg. His airway has been secured and he is being given 100% oxygen. What is the next most appropriate step in the management of this patient? A IV chlorpheniramine B IV hydrocortisone C IV saline D IV adrenaline E IM adrenaline
E
Which of the following is a cause of microcytic anaemia? A Myelodysplasia B Multiple myeloma C Thalassaemia D Myelofibrosis E Aplastic anaemia
C
A 63-year-old type 2 diabetic presents with a rash on her shins that has gradually got worse over the past 3 months. On closer inspection, there are three areas of raised, reddened and hardened skin with a yellowish centre. What is the most likely diagnosis? A Acanthosis nigricans B Diabetic dermopathy C Necrobiosis lipoidica diabeticorum D Granuloma annulare E Pruritus
C
A 76-year-old woman is brought into A&E with central crushing chest pain that radiates to her jaw and left arm. An ECG is performed, which shows ST elevation in leads ll, lll and aVF. Her SaO2 is 90%. Before she is sent to the cathlab for percutaneous coronary intervention, she is started on a combination of drugs. Which of the following should not be given? A Morphine B Oxygen C Aspirin D Clopidogrel E Warfarin
E
A 42-year-old amateur rugby player presents to his GP complaining that his team mates have been making fun of his 'man boobs'. He admits that he appears to have developed breasts over the past few months and it is causing him considerable embarrassment and distress. Which of the following drugs is most likely to have caused this unfortunate circumstance? A Cimetidine B Aspirin C Salbutamol D Ramipril E Omeprazole
A
Which part of the prostate gland undergoes progressive hyperplasia in BPH? A Central zone B Transitional zone C Peripheral zone D Ejaculatory duct E Anterior fibromuscular stroma
B
A 62-year-old obese man visits his GP complaining of a cramping pain in his buttocks that comes on when he walks his Pomeranian, Skippy. This pain was first noticed 4 months ago and it is relieved by sitting down. He has a 40 pack-year smoking history. On direct questioning, he sheepishly admits to suffering from erectile dysfunction over the last 2 months. Examination is normal except for noticeably weak pedal pulses. What is the most likely diagnosis? A Chronic compartment syndrome B Leriche syndrome C Critical limb ischaemia D Spinal stenosis E Chronic deep vein thrombosis
B
A 43-year-old man, who has recently moved to the UK from Greece, has been referred for an outpatient appointment to discuss his recurrent mouth ulcers. He says that, for the past 8 months, he has regularly suffered from oral ulcers and he, quite timidly, mentions that he has also noticed ulcers on his penis. During the consultation, you notice that his eyes look quite red. When asked about it, he mentions that his eyes have been very itchy and painful recently, but attributes this to hay fever. What is the most likely diagnosis? A Inflammatory bowel disease B Behçet’s disease C Herpes simplex virus D Syphilis E Reactive arthritis
B
Which of the following ECG findings is associated with pulmonary embolism? A Bradycardia B Tall tented T waves C Reverse tick sign D S1Q3T3 E T wave inversion
D
Which of the following is not a feature of Parkinson’s disease? A Hypomimia B Hypophonia C Micrographia D Up-gaze impairment E Wide-based gait
E
What is the inheritance pattern of polycystic kidney disease? A Autosomal dominant B Autosomal recessive C X-linked dominant D X-linked recessive E Mitochondrial
A
Which of the following statements about inflammatory bowel disease is false?
A Crohn’s disease causes transmural inflammation, whereas UC causes inflammation of the mucosa or submucosa
B Crohn’s disease causes skip lesions, whereas UC is continuous
C Crohn’s disease is associated with abscesses, fistulae, adhesions and
strictures, whereas UC is associated with toxic megacolon
D Crohn’s disease favours the rectum, whereas UC favours the terminal ileum
E Barium follow-through will show rose-thorn ulcers and a cobblestone mucosa in Crohn’s disease, but a lead pipe mucosa in UC
D
A 68-year-old care home resident is brought into A&E hyperventilating and complaining of a ‘ringing sound’ in her ears. She has a low-grade fever and appears to be confused. She has a past medical history of depression and a TIA (2 months ago). What is the most likely diagnosis? A Aspirin overdose B Paracetamol overdose C TCA overdose D Cerebrovascular accident E Pneumonia
A
A 25-year-old male presents to his GP complaining of a lump in his armpit. He says it doesn’t usually hurt except for when he goes out binge drinking with his friends. In the past few months, he has noticed that his clothes have become quite loose-fitting and he has been getting very hot and sweaty more than usual. On examination, he has firm, rubbery axillary lymphadenopathy, splenomegaly and scratch marks on his arms. A Multiple myeloma B Chronic lymphocytic leukaemia C Chronic myeloid leukaemia D Non-Hodgkin’s lymphoma E Hodgkin’s lymphoma
E
A 68-year-old man presents to his GP complaining of a cough that has been bothering him for 3 months. He says that he has coughed up large volumes of ‘rusty- coloured’ sputum. According to his hospital notes, he has been admitted 4 times in the past 12 months due to pneumonia. On examination, his fingers are clubbed and coarse crepitations are heard at the lung bases. What is the most likely underlying diagnosis? A COPD B Bronchiectasis C Pneumonia D Interstitial lung disease E TB
B
A 55-year-old woman is receiving treatment for chronic myeloid leukaemia. The consultant is concerned that this patient may have developed tumour lysis syndrome and requests some blood tests. What would you expect to see in the blood results of a patient with tumour lysis syndrome?
A LowK ,HighPO4 ,HighCa andHighUricAcid
B HighK ,HighPO4 ,LowCa andHighUricAcid
C HighK ,LowNa ,LowCa andHighMg
D LowK ,HighNa ,LowMg andHighUricAcid
E High Ca , Low PO4 and High Uric Acid
B
Which of the following is a cardiac cause of finger clubbing? A Congenital cyanotic heart disease B Viral pericarditis C Dilated cardiomyopathy D Rheumatic fever E Wolff-Parkinson-White syndrome
A
A 39-year-old homeless man is brought into A&E having been found lying in a pool of blood on the street. He is known to the A&E department having frequently been admitted for alcohol-related issues. There are no obvious signs of trauma and blood is seen in and around his mouth. Vital Signs: HR = 110 bpm; BP = 87/61 mm Hg. On examination, splenomegaly, shifting dullness and dilated veins on the anterior abdomen are identified. The registrar suspects a variceal bleed secondary to portal hypertension. What is the first step in this patient’s management?
A TIPS procedure
B Band ligation
C Terlipressin and prophylactic antibiotics
D Beta-blockers
E Terlipressin and beta-blockers
C
A 23-year-old woman comes to see her GP about some breast lumps that she has noticed over the past 6 months. She mentions that her breasts become quite painful and feel ‘lumpy’, especially in the few days before her period. The pain is relieved when she has her period. What is the most likely diagnosis? A Fibrocystic disease B Fibroadenoma C Breast cancer D Breast abscess E Duct ectasia
A
A 92-year-old female, with a history of osteoporosis, is brought into A&E by her grandson. He says that she has been drifting in and out of consciousness for the past 2 weeks and has been complaining of a headache that has been keeping her up at night and getting progressively more severe. On examination, her left pupil is dilated and displaced downwards and outwards. What investigation should be performed first? A Lumbar puncture B Carotid artery Doppler C CT head D EEG E Transthoracic echocardiogram
C
A 29-year-old man presents with a 4-day history of high fever. On inspection, you notice some needle track marks on his arms and a pansystolic murmur is heard
on auscultation, which had not previously been documented in his hospital notes. What is the most likely diagnosis?
A Mitral regurgitation
B Pericarditis
C Infective endocarditis
D Aorticstenosis
E Mitral valve prolapse
C
Which of the following is not part of the criteria for diagnosing SLE? A Pleurisy B Thrombocytopaenia C Anti-dsDNA antibodies D Oral ulcers E Heliotrope rash
E
13. A 43-year-old woman presents with a 2-month history of diarrhoea and weight loss. She has also been feeling anxious about her appearance, as many people have commented that she always looks like she is staring. On examination, her eyes appear slightly protruded and lid lag is demonstrated. She has a fine tremor in both her hands and a lumpy skin lesion is noticed on her shins. What is the most likely diagnosis? A Toxic multinodular goitre B Graves’ disease C De Quervain’s thyroiditis D Hashimoto’s thyroiditis E Riedel’s thyroiditis
B
. A 76-year-old man is admitted to hospital with a cough productive of green sputum. He has also experienced some shortness of breath and a fever. A week before his admission, his carers noted that he had a high fever, malaise and myalgia for a few days. A chest X-ray shows a cavitating lesion with an air fluid level. What is the most likely causative organism? A Staphylococcus aureus B Streptococcus pneumoniae C Legionella pneumophila D Mycoplasma pneumonia E Haemophilus influenzae
A
The urine output of a 78-year-old inpatient on the surgical ward has decreased gradually over the past 24 hours despite maintaining an adequate fluid intake. The nurses add that he has recently become rather confused and complains of nausea. U&Es are requested:
Creatinine : 231 micromol/L (baseline : 97) Urea : 12.5 mmol/L (2.5-6.7)
Na+ : 139 mmol/L (135-145)
K+ : 6.1 mmol/L (3.5-5)
An AKI is diagnosed. He is currently on ramipril (for his hypertension), bisoprolol (for his paroxysmal AF) and ibuprofen.
Which of the following steps is inappropriate in this patient’s management?
A Assess and optimise fluid status
B 10 mL of 10% calcium gluconate IV
C Stop Ramipril
D Stop bisoprolol
E Stop ibuprofen
D
A 57-year-old man is complaining of numbness and weakness in his arms. It began in his hands, 2 weeks ago, but for the last 3 days his forearms have also felt numb. On examination, there is no sensation below his elbows, tone is reduced bilaterally and the biceps and brachioradialis reflexes cannot be elicited. He adds that he recently recovered from a bout of diarrhoea and vomiting. What is the most likely diagnosis? A Multiple sclerosis B Motor neuron disease C Parkinson’s disease D Guillain-Barré syndrome E Huntington’s disease
D
A 77-year-old patient with cirrhosis presents to A&E with diffuse abdominal pain, abdominal heaviness and fever. Associated symptoms include nausea and vomiting. On examination shifting dullness is demonstrated and a fluid thrill is observed. What investigation should form part of the initial diagnostic work-up? A Abdominal X-ray B Abdominal ultrasound C Abdominal CT D Diagnostic paracentesis E Stool sample for MC&S
D
Which of the following is the correct chronological sequence of retinal changes that occur in hypertensive retinopathy?
A Papilloedema Silver Wiring Flame Haemorrhages AV Nipping
B Silver Wiring AV Nipping Flame Haemorrhages Papilloedema
C Silver Wiring Flame Haemorrhages AV Nipping Papilloedema
D AV Nipping Papilloedema Silver Wiring Flame Haemorrhages
E AV NippingSilver wiringPapilloedemaFlame Haemorrhages
B
A 44-year-old woman is complaining of pain and a tingling feeling in the lateral half of her right hand. She often finds that she wakes up in the middle of the night because of the pain, which is then relieved by shaking her hand vigorously. Which nerve has been affected? A Ulnar nerve B Radial nerve C Musculocutaneous nerve D Median nerve E Posterior interosseous nerve
D
Which of the following full blood count and clotting screen results is consistent with a diagnosis of disseminated intravascular coagulation?
A High platelets, High Hb, High APTT/PT, High fibrinogen
B High platelets, High Hb, Low APTT/PT, High fibrinogen
C Low platelets, High Hb, Low APTT/PT, Low fibrinogen
D Low platelets, Low Hb, High APTT/PT, Low fibrinogen
E Low platelets, Low Hb, Low APTT/PT, Low fibrinogen
D
A 40-year-old woman is admitted to A&E with shortness of breath that began suddenly a day after she returned from a holiday to the Maldives. What is the first step in her management?
A D-dimer
B High flow oxygen and low molecular weight heparin
C IV heparin
D CTPA
E Venous ultrasound of the lower limbs
B
What is the gold standard diagnostic test for acromegaly? A Insulin suppression test B Oral glucose tolerance test C Short synacthen test D IGF-1 levels E Thyroid function test
B
Which of the following sets of results would be consistent with alcoholic hepatitis?
A Elevated MCV, ALT:AST > 2 and elevated GGT
B Elevated MCV, AST:ALT > 2 and elevated GGT
C Reduced MCV, AST:ALT >2 and elevated GGT
D Elevated MCV, ALT:AST > 2 and reduced GGT
E Reduced MCV, AST:ALT > 2 and reduced GGT
B
An inpatient on the orthopaedic surgery ward has recently developed a cough, high fevers and chills. Blood cultures are taken which identify MRSA. Which of the following antibiotics is often used in the treatment of MRSA infections? A Vancomycin B Flucloxacillin C Tazocin D Metronidazole E Cefuroxime
A
A 77-year-old man is referred to the outpatient clinic by his GP having presented with chest pain and worsening shortness of breath. He has a history of COPD, diagnosed 12 years ago. On examination, his JVP is elevated, a parasternal heave is palpated and auscultation reveals an early diastolic murmur. An ECG is performed which shows right-axis deviation, a tall R wave in V1 and peaked P waves in lead ll. What is the most likely diagnosis? A Aortic regurgitation B Mitral stenosis C Pulmonary hypertension D Right heart failure E Exacerbation of COPD
C
Which of the following is not a histopathological type of malignant melanoma? A Superficial spreading B Acral lentiginous C Bowen’s disease D Nodular E Lentigo maligna
C
An 82-year-old man has recently suffered from a right-sided stroke and is undergoing physiotherapy. He is referred for an upper limb neurological examination. The power in his right arm is normal. He can abduct his left arm by himself, but fails to maintain that position as soon as any resistance is applied. What is the MRC grading of his left shoulder abduction? A Grade 1 B Grade 2 C Grade 3 D Grade 4 E Grade 5
C
A 54-year-old female, with a BMI of 28, presents with a 2-year history of epigastric pain that radiates to the neck. It gets worse when lying down, and she also complains of painless regurgitation of food. What is the most appropriate investigation to confirm the diagnosis? A Chest X-ray B Barium swallow C ECG D OGD E Manometry
B
Which of the following is not an indication for dialysis in the context of acute kidney injury? A Refractory hyperkalaemia B Refractory pulmonary oedema C Uraemic pericarditis D Severe metabolic acidosis E Macroscopic haematuria
E
A 25-year-old man, with no past medical history, presents to A&E with sudden- onset shortness of breath and right-sided chest pain, which started whilst he was playing football. Vital Signs: RR = 24 /min; HR = 125 bpm; BP = 85/59 mm Hg. There is a hyper-resonant percussion note and reduced breath sounds over the right upper zone and the trachea is deviated to the left. A chest X-ray confirms the presence of a pneumothorax measuring 3 cm. What is the most appropriate management option?
A Give analgesia and reassure
B Admit to hospital, monitor vital signs and repeat chest X-ray in 3 hours
C Insert a chest drain
D Insert a large-bore cannula into the right 2nd intercostal space in the
midclavicular line
E Surgical pleurectomy
D
Which of the following organisms is a recognised cause of hospital-acquired pneumonia? A Streptococcus pneumoniae B Pseudomonas aeruginosa C Haemophilus influenzae D Legionella pneumophila E Chlamydophila psittaci
B
A 19-year-old girl visits her GP after experiencing painful urination over the past week. She has also been urinating more frequently than usual and complains that her urine looks cloudy and smells particularly bad. A urinary tract infection is suspected. Which investigation can definitively confirm the diagnosis? A Urine dipstick B CRP C Blood cultures D MSU E U&Es
D
. A 31-year-old man presents with a 2-day history of diffuse watery diarrhoea and nausea. He admits to recently eating a BBQ at a friend’s house. How should this patient be managed?
A Bed rest and oral rehydration
B Bed rest, oral rehydration and antibiotics
C Anti-diarrhoeal agents
D Call an ambulance and admit to hospital
E Refer to a gastroenterologist for further investigation
A
- A 16-year-old boy presents to his GP after noticing the growth of several small, fleshy tags on his torso. He has also noticed that he has many more ‘birthmarks’ now compared to when he was younger. On examination, there are 8 coffee-coloured, flat
skin lesions (‘birthmarks’) which are about 2-3 cm in diameter. Freckling around both axillae is also noted. What is the most likely diagnosis?
A Tuberous sclerosis
B Neurofibromatosis type 1
C Neurofibromatosis type 2
D Xeroderma pigmentosum
E Dercum disease
B
Which of the following is an extra-articular feature of ankylosing spondylitis? A Erythema ab igne B Subcutaneous nodules C Apical lung fibrosis D Mitral regurgitation E Tophi
C
A 43-year-old man has been involved in a bar fight. He is rushed, unconscious, into A&E with a stab wound to the chest. On examination, his JVP is raised and he is hypotensive (BP : 86/70 mm Hg). Auscultation of his chest reveals very quiet heart sounds. What is the most likely diagnosis? A Acute heart failure B Haemopericardium C Pneumothorax D Septic shock E Hypovolaemic shock
B
A 52-year-old man attends the respiratory clinic complaining of a dry cough that has been bothering him for 2 months. He has never smoked and has not experienced any shortness of breath or chest pain. Respiratory examination detects no abnormalities. He has a past medical history of hypertension, for which he started treatment 4 months ago. What is the most likely diagnosis? A Asthma B Interstitial lung disease C Bronchiectasis D Drug side-effect E Atypical pneumonia
D
A 41-year-old female is referred to the dermatology clinic because she has developed multiple purple nodules on her shins. They are tender and have a diameter of 1-2 inches. Erythema nodosum is suspected. Which of the following is not a cause of erythema nodosum? A Tuberculosis B Reaction to sulphonamides C Inflammatory bowel disease D Ankylosing spondylitis E Behçet’s disease
D
A 49-year-old woman is admitted to A&E complaining of severe right upper quadrant pain that began last evening and has not subsided. She admits to eating a lot of fast food and mentions that, in the past, she has experienced a stabbing pain for a couple of hours after eating. The pain during these episodes is less intense than the pain she is currently experiencing, and it tends to be localised around her epigastrium. She drinks no more than 12 units of alcohol per week and has not lost any weight recently. On examination, she is jaundiced and Murphy’s sign is positive. LFTs are requested: Bilirubin : 45 micromol/L (3-17) AST : 50 iU/L (5-35) ALT : 45 iU/L (5-35) ALP : 400 iU/L (30-150) What is the most likely diagnosis? A Gallstones B Alcoholic hepatitis C Viral hepatitis D Hepatocellular carcinoma E Gilbert’s syndrome
A
A 34-year-old Nigerian man brings his 3-year-old son to A&E. He has been crying and complaining of severe pain in his hands. On examination, his fingers are swollen and warm. The junior doctor suspects a painful crises of sickle cell disease. What is the mode of inheritance of sickle cell disease? A Autosomal recessive B Autosomal dominant C X-linked recessive D X-linked dominant E Y-linked
A
A 16-year-old girl is rushed to A&E by her parents. She is unconscious and shaking uncontrollably. Her mother says that she has been fitting for over half an hour. What is the most appropriate first step in the management of this patient?
A IV phenytoin
B IV thiopentone
C IV lorazepam/PR diazepam
D Reassure her parents and let the seizure terminate by itself
E Oral sodium valproate
C
A 61-year-old, with a 40 pack-year smoking history, presents with confusion. He has also had a 3-month history of weight loss and haemoptysis. A blood test shows the following results: Na+ : 121 mmol/L (135 - 145) K+ : 4.1 mmol/L (3.5 - 5) Ca2+ : 2.3 mmol/L (2.2-2.6) What is the most likely diagnosis? A Addison’s disease B Hypothyroidism C Heart failure D SIADH E Cirrhosis
D
A 27-year-old man presents complaining of sharp chest pain. He mentions that he has taken a few days off work recently because of the flu. What would you expect to see on his ECG?
A ST elevation in leads II, III and aVF
B Widespread saddle-shaped ST elevation
C ST depression
D Tented T waves
E Absent P waves
B
Why are urinary tract infections more common in women?
A Women have a shorter urethra
B Men have a larger bladder
C Men have a shorter distance between their urethral opening and their anus
D Women are more likely to be catheterised
E Men have longer ureters
A
A 44-year-old bus driver from the West Indies has suffered from shortness of breath and a dry cough for the last 4 months. He also complains of some ‘sore lumps on his shins’. Closer inspection reveals tender violet nodules on both shins. A chest X-ray is requested, which shows bilateral hilar lymphadenopathy. Blood tests are also requested, including U&Es – which parameter would you expect to be raised? A Sodium B Potassium C Calcium D pH E Urea
C
A 35-year-old woman presents to clinic with a 6-month history of watery diarrhoea and abdominal pain that improves after defecation. She admits to defecating 4-5 times per day compared to her normal frequency of once per day. On examination, a papulovesicular rash is seen on both elbows. Which investigation would be most useful in aiding the diagnosis? A Stool sample for MC&S B Serology for anti-tTG antibodies C Full blood count D Blood cultures E Barium follow-through
B
Which of the following is a risk factor for breast cancer? A Breastfeeding B Late menarche C Early menopause D Not having children E Age<50yrs
D
A 26-year-old man presents with an acutely swollen left knee. The pain and swelling started 2 days ago, but he didn’t pay much attention to it as he thought it was just a muscle strain. He has also developed a fever over the past 24 hours. On examination, his left knee is red, swollen and extremely painful on passive flexion. Septic arthritis is suspected and a joint aspirate is requested. Which of the following organisms most commonly causes this condition? A Haemophilus influenza B Staphylococcus aureus C Neisseria meningitidis D Escherichia coli E Mycobacterium tuberculosis
B
A 44-year-old female presents to her GP complaining of worsening hearing. A full cranial nerves examination is performed. When Weber’s test is performed, she hears the sound louder in her left ear. Then, Rinne’s test is performed and she reports that, in both ears, the sound is loudest when the tuning fork is held in front of the auditory canal rather than when the fork is held against her mastoid processes. Which of the following best describes the patient’s condition?
A Conductive hearing loss in the right ear
B Conductive hearing loss in the left ear
C Sensorineural hearing loss in the right ear
D Sensorineural hearing loss in the left ear
E Bilateral sensorineural hearing loss
C
Which of the following best defines chronic kidney disease?
A GFR < 60 mL/min/1.73m2 for more than 3 months
B GFR < 60 mL/min/1.73m2 for more than 6 months
C GFR < 90 mL/min/1.73m2 for more than 3 months
D GFR < 90 mL/min/1.73m2 for more than 6 months
E Requirement of long-term renal replacement therapy
A
A 77-year-old man, with a history of ischaemic heart disease, presents to A&E with acute-onset dyspnoea, a wheeze and a cough productive of pink frothy sputum.
A diagnosis of acute left ventricular failure, resulting in pulmonary oedema, is made. Which of the following is not part of the immediate management of this patient?
A Administer oxygen
B Lie the patient down
C IV Diamorphine
D GTN infusion
E IV furosemide
B
Which of the following is not a cause of macrocytic anaemia? A Iron deficiency B Vitamin B12 deficiency C Folate deficiency D Methotrexate E Hypothyroidism
A
A 62-year-old shop owner is brought to A&E, by his daughter, having experienced worsening shortness of breath. His face and arms have also become quite swollen. On examination, he has a plethoric face and his JVP is raised and non-pulsatile. He seems disinterested when the history is taken because he does not trust doctors. His daughter adds that he has been coughing up blood and losing weight for about 6 months, however, he has refused to seek medical attention until his recent worsening of symptoms. What is the most likely diagnosis? A Asbestosis B Congestive cardiac failure C Polycythaemia D Superior vena cava syndrome E Mesothelioma
D
- An 82-year-old man is brought to A&E with a severe headache. He complains of an intense aching pain, focused around his right eye, that has rapidly worsened. He has never experienced anything like this before. A nurse informs you that he vomited whilst waiting to be admitted, and has been complaining of nausea since. Closer inspection reveals a red, congested right eye with a cloudy cornea. He complains that his vision has worsened with the onset of this headache and he has started
seeing haloes around all sources of light. What is the most likely diagnosis?
A Meningitis
B Subarachnoid haemorrhage
C Acute glaucoma
D Cluster headache
E Migraine
C
Which of the following is associated with left ventricular systolic failure? A Pulsus alternans B Pulsus paradoxus C Water-hammer pulse D Pulsus parvus et tardus E Pulsus bisferiens
A
26. A 46-year-old truck driver is accompanied by his wife to see his GP after he fell asleep at the wheel two days ago. He appears to be quite shaken by the ordeal as he recalled having to veer away from oncoming traffic. On further questioning, he says that his sleep hasn’t been disrupted, however, he has been feeling very tired during the day. His wife interjects and mentions that her own sleep has been disturbed because her husband has been ‘snoring ferociously’. When asked about diet and exercise, he admits to eating badly and exercising very little since he started working as a truck driver 3 years ago. In that time, he has gained a considerable amount of weight. What is the most likely diagnosis? A Narcolepsy B Cataplexy C Absence Seizure D Obstructive Sleep Apnoea E Central Sleep Apnoea
D
A 28-year-old IV drug-user visits his GP practice for an annual check-up. Hepatitis serology is requested and the following results are reported. HBsAg - HBeAg - HBcAb IgM - HBcAb IgG + HBsAb + What is the hepatitis status of this patient? A Acute infection B Chronic infection C Cleared D Vaccinated E Susceptible
C
A 24-year-old student presents at his GP practice with a 2-day history of blood in his urine. Urine dipstick reveals proteinuria and haematuria. On questioning, he mentions that he has been recovering from a sore throat and a cough over the last 4 days. What is the most likely diagnosis?
A Minimal change disease
B IgA nephropathy
C Membranous nephropathy
D Post-streptococcal glomerulonephritis
E Acute tubular necrosis
B
Which of the following is not a feature of Cushing’s syndrome? A Central obesity B Poor wound healing C Hypotension D Striae E Proximal myopathy
C
A 24-year-old female is brought to A&E having fallen off a stage in a nightclub whilst under the influence of LSD. Her eyes open when the registrar squeezes her trapezius and she makes a few incomprehensible sounds. Her arms flex, wrists clench and legs extend and internally rotate in response to pain. What is her GCS? A5 B7 C8 D9 E 10
B
An 8-year-old girl visits her GP, with her mother, 4 weeks after being prescribed antibiotics for a sore throat. Her urine has become tea-coloured and she has been feeling nauseous with a headache. Urinalysis reveals proteinuria and haematuria and her blood pressure is 137/72 mm Hg. A diagnosis of post-infectious glomerulonephritis is suspected. Which organism is most likely responsible? A Streptococcus agalactiae B Streptococcus pyogenes C Escherichia coli D Diphtheria E Bordetella pertussis
B
Which of the following drugs is not used in the long-term management of chronic heart failure? A Indomethacin B Carvedilol C Spironolactone D Candesartan E Digoxin
A
A 32-year-old man has been suffering from worsening shortness of breath over the past 5 months. He used to be very physically active, but, recently, he has become breathless whilst doing relatively low intensity tasks such as walking to the bus stop. He has also developed a chronic cough, productive of clear sputum. He regularly visits the hospital to monitor his liver function because of a ‘liver disease’ that he has had since he was a child. He has never smoked and does not drink alcohol. What is the most likely diagnosis? A COPD B Autoimmune hepatitis C 1 antitrypsin deficiency D Haemochromatosis E Wilson’s disease
C
A 55-year-old woman presents with a 2-month history of jaundice and right upper quadrant pain. She has a history of ulcerative colitis. LFTs and serology are requested: ALP : 390 iU/L (30-150) AST : 40 iU/L (5-35) ALT : 40 iU/L (5-35) GGT : 150 iU/L (7-32) pANCA : Positive What is the most likely diagnosis? A Haemochromatosis B Primary biliary cirrhosis C Primary sclerosing cholangitis D Autoimmune hepatitis E Wilson’s disease
C
- An 89-year-old woman is brought into A&E after she suddenly became very disorientated and was unable to recognise her own son. On examination, she is blind
in the left half of her visual field. An ischaemic stroke is suspected. Which artery is most likely to be involved?
A Right anterior cerebral artery
B Right posterior cerebral artery
C Left posterior cerebral artery
D Right middle cerebral artery
E Left middle cerebral artery
B
Which of the following conditions is strongly associated with Giant Cell Arteritis? A Takayasu’s aortitis B Myalgic encephalomyelitis C Fibromyalgia D Polymyalgia rheumatic E Polymyositis
D
A 58-year-old woman presents with a scaly rash around her right nipple. She says that the rash has been there for the last 3 weeks. On examination, there is a crusty rash around her right nipple and palpation reveals a firm lump just below the areola that appears to be tethered to surrounding tissues. What is the most likely diagnosis?
A Intraductal papilloma
B Phyllodes tumour
C Paget’s disease of the breast D Breast abscess
E Fibroadenoma
C
A 43-year-old businesswoman has had a TIA. Soon after landing in the UK from a business trip to Australia, she suddenly became unable to move her right arm. She began slurring her speech, the right side of her face started drooping and she temporarily lost vision in her right eye. She adds that she experienced some pain in her right leg as she was disembarking the plane, however, she assumed it was a muscle strain from wearing high-heels for several days. What underlying defect is most likely to have caused her TIA? A Atrial fibrillation B Atrial septal defect C Carotid atherosclerosis D Infective endocarditis E Prosthetic heart valve
B
Which of the following is not a recognised cause of acute pancreatitis? A Hyperlipidaemia B Hypothermia C Hypocalcaemia D Mumps E Azathioprine
C
A 34-year-old man is brought to A&E having collapsed in a shopping mall. He did not lose consciousness but mentioned that he felt dizzy and could feel his ‘heart racing’. He has a past medical history of asthma. His ECG shows a regular narrow
complex tachycardia with no visible P waves. Vagal manoeuvres failed to terminate the tachycardia. What is the next most appropriate step in the management of this patient?
A Verapamil
B Amiodarone
C Adenosine
D Bisoprolol
E Flecainide
A
Which of the following is a cause of primary amenorrhoea? A Prolactinoma B Pregnancy C Haemochromatosis D Polycystic ovarian syndrome E Turner syndrome
E
In which part of the nephron does bendroflumethiazide have its effect? A Proximal convoluted tubule B Descending limb of the loop of Henle C Ascending limb of the loop of Henle D Distal convoluted tubule E Collecting duct
D
A 62-year-old heavy smoker is being investigated for lung cancer having presented with a 4-month history of unintentional weight loss, haemoptysis and fatigue. He claims that his voice has become hoarse and a junior doctor adds that he has a textbook ‘bovine cough’. In which part of the lung is the tumour most likely to be found? A Left apex B Right middle lobe C Right base D Left base E Pleura
A
Which of the following most accurately describes the sodium and potassium requirements of a 70 kg man over a 24 hour period? A 100 mmol Na+ & 40-50 mmol K+ B 120 mmol Na+ & 5-10 mmol K+ C 120 mmol Na+ & 10-20 mmol K+ D 120 mmol Na+ & 60-70 mmol K+ E 140 mmol Na+ & 60-70 mmol K+
E
A 27-year-old female presents to her GP complaining of an episode of painful loss of vision that lasted 1 day and resolved spontaneously. 6 months ago, she lost sensation across the lateral half of her left leg which also resolved spontaneously. Which of the following would you expect to see in her diagnostic work up?
A Bence Jones proteins
B Oligoclonal bands on CSF electrophoresis
C High CSF protein
D Xanthochromia
E Raised ICP
B
Which of the following is an acquired cause of long QT syndrome? A Romano-Ward syndrome B Hyponatraemia C Hyperkalaemia D Hypomagnesaemia E Hypercalcaemia
D
A 70-year-old man presents with a 3-month history of polyuria. He has been urinating up to 12 times per day and has also experienced some constipation, abdominal pain and back pain. More recently, he has noticed that his face appears ‘puffier’ than usual and his ankles are swollen. He is on citalopram to treat his depression and takes no other regular medications. Blood tests reveal: ESR = 64 mm/hr (< 22). Urinalysis reveals: Protein: Positive Blood: Negative 24 hr urine protein (g): 9.8 (< 3.5) What is the most likely diagnosis? A Cushing’s syndrome B Amyloidosis C Glomerulonephritis D Malignancy E Congestive cardiac failure
B
What is the most common cause of urinary tract infections? A Staphylococcus aureus B Staphylococcus saprophyticus C Enterococcus faecalis D Escherichia coli E Klebsiella pneumonia
D
A 36-year-old man presents to his GP with a 1-month history of shortness of breath on exertion. He has also experienced a low-grade fever and a dry cough. He has a past medical history of HIV. A pulse oximeter is attached showing an oxygen saturation of 97% at rest. The patient is then asked to walk up and down the room a few times and his oxygen saturation drops to 88%. What is the most likely diagnosis? A Interstitial lung disease B Pulmonary embolism C Pneumocystis jirovecii pneumonia D Mycobacterium avium complex E Bronchiectasis
C
An 88-year-old care home resident is receiving oral clarithromycin to treat a chest infection. She develops profuse watery diarrhoea and her temperature rises to 38.2C. She has vomited three times and is experiencing diffuse abdominal
discomfort. A stool sample is positive for Clostridium difficile toxin. Which antibiotic should be given to this patient?
A Co-amoxiclav
B Penicillin
C Metronidazole
D Ciprofloxacin
E Tetracycline
C
What is another name for target cells? A Codocyte B Dacrocyte C Spherocyte D Reticulocyte E Schistocyte
A
A 34-year-old man with Marfan’s syndrome, comes to A&E having experienced a sudden tearing chest pain. He adds that the pain seems to move to his back. On examination, an early diastolic murmur is heard over the aortic valve and unequal arm pulses are palpated. What is the most likely diagnosis? A Ruptured aortic aneurysm B Coarctation of the aorta C Aortic dissection D Myocardial infarction E Tension pneumothorax
C
Which set of spirometry results is most likely to be seen in a patient with COPD?
A FEV1 > 0.8 and FEV1:FVC > 0.7
B FEV1 < 0.8 and FEV1:FVC < 0.7
C FEV1 > 0.8 and FEV1:FVC < 0.7
D FEV1 < 0.8 and FEV1:FVC > 0.7
E Impossible to tell without FVC measurement
B
A 15-year-old boy arrives at A&E with sudden-onset pain and swelling in his scrotum, which began 2 hours ago whilst playing a rugby match. He also starts vomiting and complains of pain in his right iliac fossa. On examination, his right hemiscrotum is red and swollen. What is the most appropriate first step in his management? A Doppler ultrasound of the testes B CT Scan C Exploratory surgery D Empirical antibiotics E Abdominal X-ray
C
Which scoring system is used to determine a patient’s risk of developing pressure sores? A GRACE score B ABCD2 score C Ranson score D W aterlow score E Rockall score
D
A 36-year-old man presents with a 3-week history of fatigue, frequent urination and excessive thirst. He also mentions that he has been unable to take part in his weekly 5-a-side football sessions for the past month because his ‘muscles feel weak’. The patient’s notes reveal that, during a previous appointment 6 months ago, his blood pressure was measured at 164/98 mm Hg. He was offered a follow-up appointment to discuss management options, however, he did not attend. The GP measures the patient’s blood pressure again, and it is 172/102 mm Hg. What would you expect to see on the ECG of this patient? A Tented T waves B Absent P waves C ST elevation D J waves E U waves
E
. A 40-year-old man is brought into A&E after he was found lying unconscious on the side of the road with an empty bottle of whisky next to him. Once he regains consciousness, he starts yelling at the ward staff expressing that he thinks he has been kidnapped and is being held hostage. He jumps out of bed, but finds it difficult to walk. He has a wide-based gait and he is taking small steps. Given the most likely diagnosis, what should form part of the immediate management? A Acamprosate B Chlorediazepoxide C Thiamine D Naloxone E Disulfiram
C
Which of the following is not a reversible cause of cardiac arrest? A Hypothermia B Tension pneumothorax C Cardiac tamponade D Hypokalaemia E Pleurisy
E
A 44-year-old woman presents with right upper quadrant pain that radiates to the tip of her right shoulder with an intermittent fever, severe chills and sweats. She has also recently experienced hiccups, breathlessness and a dry cough. On examination, there is tenderness over the 8th to 11th ribs on the right side and dullness to percussion, diminished breath sounds and reduced chest expansion over the lower zone of the right lung. 10 days prior to the onset of symptoms, she had a laparoscopic appendectomy. What is the most likely diagnosis? A Acute cholangitis B Basal pneumonia C Subphrenic abscess D Atelectasis E Liver abscess
C
Which of the following endocrine conditions can cause hyperprolactinaemia? A Cushing’s syndrome B Phaeochromocytoma C Addison’s disease D Graves’ disease E Hypothyroidism
E
An 18-year-old student is brought to see his GP by his mother. Since returning from university 2 days ago, he has become drowsy and has developed a fever. He has also vomited 3 times. On examination, the patient complains of pain when the GP flexes his hip and extends his knee, and a non-blanching rash is seen on his trunk. What should the GP do next?
A Reassure and discharge
B Administer IV or IM benzylpenicillin and call an ambulance
C Administer IV dexamethasone and call an ambulance
D Prescribe oral benzylpenicillin and arrange a follow up appointment
E Prescribe oral dexamethasone and arrange a follow up appointment
B
A 24-year-old man presents to the outpatient clinic with a 3-month history of lower abdominal pain and bloody diarrhoea. He often defecates more than 4 times per day and sometimes does not feel completely empty afterwards. On examination, his fingers are clubbed and a large irregular ulcer is found on his left shin. What is the most likely diagnosis? A Irritable bowel syndrome B Gastroenteritis C Crohn’s disease D Ulcerative colitis E Coeliac disease
D
Which joint is most commonly affected in gout? A 1st Metacarpophalangeal joint B 1st Metatarsophalangel joint C 1st Tarsometatarsal joint D 1st Interphalangeal joint E Talonavicular joint
B
A 10-year-old boy is brought to the respiratory clinic by his mother. Since he was very young, he has suffered from recurrent infections and has been hospitalized many times. As he has frequently moved country, a formal diagnosis has never been made. In the past 6 months, he has become increasingly breathless and has
experienced a chronic cough, productive of large volumes of purulent sputum. A chest X-ray is performed revealing widespread bronchiectasis, and situs inversus. What is the most likely diagnosis?
A Cystic fibrosis
B Young’s syndrome
C Kartagener’s syndrome
D Caplan’s syndrome
E 1 antitrypsin deficiency
C
Which system is used to stage Hodgkin’s lymphoma? A Ann Arbor B Gleason C Dukes’ D Rai and Binet E Breslow
A
A 63-year-old woman presents with a moist, shallow ulcer just superior to the medial malleolus of her left foot. It is diagnosed as a venous ulcer. Which of the following features is not associated with venous ulcers? A Varicose veins B Calloused edges C Stasis eczema D Haemosiderin deposition E Lipodermatosclerosis
B
Which of these conditions does not typically cause eye signs? A Ulcerative colitis B Crohn’s disease C Ankylosing spondylitis D Reactive arthritis E Cervical spondylosis
E
What is the most common cause of encephalitis in the UK?
A Herpes simplex virus B Syphilis C EBV D Varicella zoster virus E Coxsackie virus
B
Whilst enjoying a drink in a bar with some friends, a 36-year-old woman feels a sudden sensation of tingling in her ring finger which spreads to the rest of her hand over a couple of seconds. She only feels the sensation in her right hand and she maintains awareness throughout the episode, which lasts less than a minute. What type of seizure is this describing? A Absence B Simple partial C Complex partial D Myoclonic E Atonic
B
An 82-year-old lady has been in hospital for 4 weeks due to a hip fracture that she sustained after falling at home. Due to her immobility, a pressure sore has developed on the heel of her right foot. There is an intact fluid-filled blister measuring 3 inches in diameter. The ulcer is superficial and there is no subcutaneous tissue visible. According to the EPUAP, what grade of severity is this pressure ulcer? A Grade 1 B Grade 2 C Grade 3 D Grade 4 E Ungradable
A
A 49-year-old bird keeper has become more and more breathless over the last 6 months. She used to be able to easily complete her daily dog walk, however, recently, she has found that she is having to taking more breaks to catch her breath. She has also had a dry cough. Examination reveals fine inspiratory crackles and a chest X-ray shows reticulo-nodular shadowing. What is the most likely diagnosis? A Extrinsic allergic alveolitis B COPD C Pneumoconiosis D Aspergillosis E Asbestosis
A
A 64-year-old man has been referred for an outpatient appointment with the urology department. Over the past 5 months, he has been urinating around 10-12 times per day. He often takes several minutes to start urinating and his stream is much weaker than it used to be. Once he has finished, he does not feel ‘completely empty’ and finds that he ‘leaks a little bit’ as well. Digital rectal examination reveals a smoothly enlarged prostate gland with a palpable midline sulcus. A diagnosis of benign prostatic hyperplasia is made. He is eager to avoid surgery if possible. Which treatment would be best for him? A Oxybutynin B Solifenacin C Tamsulosin D Nitrofurantoin E Co-trimoxazole
C
A 53-year-old lady has been feeling increasingly short of breath over the past 6 months. She adds that she always feels very tired and has, more recently, experienced a tingling sensation in both hands. Neurological examination reveals a sensory neuropathy affecting only the hands and feet. What is the most likely cause of her symptoms? A Iron deficiency anaemia B Anaemia of chronic disease C Folic acid deficiency D Vitamin B12 deficiency E Thalassemia trait
D
- A 33-year-old female with SLE presents to the fertility clinic complaining of repeated miscarriages. She has been desperately trying to start a family but has
unfortunately suffered 3 miscarriages over the last 7 years. Her past medical history includes an appendicectomy (aged 12) and two DVTs. Given the likely diagnosis, which of the following antibodies is associated with this disease?
A Anti-CCP antibody
B Anti-Jo-1 antibody
C Anti-centromere antibody
D Anti-cardiolipin antibody
E Anti-smooth muscle antibody
D
Which of the following is not associated with infective endocarditis? A Clubbing B Janeway lesions C Rose spots D New pansystolic murmur E Splinter haemorrhages
C
A 43-year-old woman presents with a ‘rather embarrassing’ problem. Since the birth of her fourth child, 3 months ago, she has wet herself several times. She has noticed that whenever she laughs or coughs, a little bit of urine leaks out without her control. What is the name of this type of incontinence? A Functional incontinence B Stress incontinence C Urge incontinence D Overflow incontinence E Double incontinence
B
A 44-year-old man has been suffering from constant, nagging headaches for the past 2 years. In this time, he has also noticed that his hands and feet appear to have grown as he has changed shoe size 3 times, and had to get his wedding ring cut off. He has been relatively healthy throughout his life except for undergoing surgery for carpal tunnel syndrome last year. What is the most likely diagnosis? A Cushing’s disease B Acromegaly C Hypothyroidism D Gigantism E Prolactinoma
B
Which of the following is used to predict the severity of acute pancreatitis? A Alvarado score B Rockall score C Modified Glasgow score D Glasgow-Blatchford score E Child-Pugh score
C
A 31-year-old scuba diving instructor, living in the Maldives, had a seizure three days ago. He has no history of epilepsy, however, he has had persistent headaches over the past 5 months. He adds that the headaches are particularly bad when he goes to bed. On examination, a dark, irregular skin lesion is found on the back of his neck. An MRI scan shows multiple lesions across both cerebral hemispheres. What is the most likely diagnosis? A Glioblastoma multiforme B Metastases C Neurofibromatosis Type 1 D Acoustic neuroma E Meningioma
B
Which of the following examination findings is not associated with COPD? A Use of accessory muscles B Breathing through pursed lips C Peripheral cyanosis D Clubbing E Bounding pulse
D
A 56-year-old man comes to A&E with a very swollen glans. He went to the toilet to urinate last night, however, once he had finished, he was unable to replace his foreskin back over his glans. Since then, his glans has gradually become very painful and inflamed. What is the name of this condition? A Phimosis B Paraphimosis C Balanitis D Priapism E Peyronie’s disease
B
A 59-year-old man presents to A&E with fatigue and shortness of breath, that gets worse when lying down. He has also been coughing up pink, frothy sputum. An echocardiogram is performed, which shows aortic regurgitation. On closer inspection
of the patient’s hands, his nail beds appear to be pulsating. What is the name of this sign? A de Musset’s sign B Quincke’s sign C Traube’s sign D Corrigan’s sign E Becker’s sign
B
A 61-year-old man attends an outpatient clinic appointment complaining of epigastric pain that gets better soon after eating. A urease breath test confirms the presence of H. pylori. A duodenal ulcer is suspected. What is the most appropriate management option?
A 1 week of once daily omeprazole, amoxicillin and clarithromycin
B 1 week of twice daily omeprazole, amoxicillin and clarithromycin
C 1 week of once daily ranitidine, amoxicillin and clarithromycin
D 1 week of twice daily ranitidine, amoxicillin and clarithromycin
E 1 week of once daily omeprazole, ranitidine and amoxicillin
B
Which of the following is a major consequence of folate deficiency in pregnancy? A Incomplete limb development B Neural tube defects C Congenital cardiac abnormalities D High birth weight E Cleft palate
B
Which of the following is true about the target population group and frequency of breast cancer screening in the UK? A Women aged 35-65 every 3 years B Women aged 40-60 every 3 years C Women aged 40-70 every 5 years D Women aged 50-75 every 5 years E Women aged 50-70 every 3 years
E
A 46-year-old woman is brought to A&E complaining of severe right upper quadrant pain and a high fever with rigors. On examination, she is jaundiced, febrile and tachycardic. Vital Signs: RR = 24 breaths per minute, HR = 112 bpm, Temp = 38.9°C. A full blood count is requested: Hb = 142 g/L (115-160) WBC = 14.7 x 109/L (4-11) Platelets = 370 x 109/L (150-400) She is diagnosed with ascending cholangitis and her disease is managed using ‘The Sepsis Six’ protocol. Which of the following is not part of ‘The Sepsis Six’? A Give high-flow oxygen B Take blood smear C Give IV antibiotics D Measure urine output E Measure serum lactate
B
A 15-year-old girl is brought, by her mother, to see her GP. She is concerned that her daughter has been acting ‘very weird’ over the past few months. Having previously been quite shy and reserved, she has recently had several rude outbursts towards her parents. In addition, her performance at school has deteriorated. On examination, she appears slightly jaundiced. Closer inspection of her eyes using a slit-lamp shows dark rings around her iris. Given the most likely diagnosis, which of the following would you expect to see in the diagnostic work up? A Low serum caeruloplasmin B High serum copper C Low AST D Low ALP E High transferrin saturation
A
A 4-year-old boy, who has recently moved to the UK from Cameroon, has been suffering from frequent infections and breathing difficulties since he was born. His mother tells you that he always has a cough and regularly suffers from chest infections. He has also had some bowel problems – his stools are often loose and irregular. On examination, he appears small for his age, his fingers are clubbed and bilateral coarse crackles are heard on auscultation. Cystic fibrosis is suspected. Which investigation should be requested to confirm the diagnosis? A Chest X-ray B Sweat test C Faecal elastase D Sputum culture E Stool culture
B
Which of the following is not a risk factor for the formation of a DVT? A Factor V Leiden B Malignancy C Nephrotic syndrome D Antiphospholipid syndrome E Alcohol
E
A 77-year-old retired ship-builder presents to his GP complaining of a 4-month history of right-sided chest pain, shortness of breath and 4 kg of weight loss. A chest X-ray is requested, which shows an ill-defined mass at the right pleural margin. What is the most likely diagnosis? A Small cell lung cancer B Adenocarcinoma C Mesothelioma D Squamous cell lung cancer E Large cell lung cancer
C
In the treatment of acute coronary syndrome (ACS), which of the following statements is FALSE?
A Aspirin and clopidogrel do not provide enough anticoagulation; heparin should also be given
B Give 75 mg aspirin stat
3. C Give 300 mg clopidogrel in addition to aspirin
D Hypotension, asthma and bradycardia are the main contraindicators to
beta blockade
E Patients will likely continue taking a statin, beta-blocker and angiotensin-
converting enzyme (ACE) inhibitor on discharge home
B
Which of the following is not a potential cause of obstructive renal impairment?
A Benign prostatic hypertrophy B Recurrent kidney stones
C Retroperitoneal fibrosis
D Schistosomiasis
E Systemic sclerosis
E
A 60-year-old man who works for an oil company presents with a lesion on the temple that is bothering him as it is growing. It bled once when he knocked it. On examination, the lesion is 8 mm in diameter and is a flat, mildly erythematous patch with a few scales and a larger keratotic horn in the centre. There are no other lesions on inspection of his skin and no personal or family history of skin cancer.
Which of the following is the most appropriate management plan?
A Cryotherapy
B Curettage
C Excisional biopsy
D Topical 5-fluorouracil
E Wide local excision
A
A 68-year-old man who is recently diagnosed with lung cancer is admitted to the emergency department with acute shortness of breath. A chest X-ray shows a right upper zone (RUZ) collapse.
What do you expect to find on examination?
Trachea
A Deviated to right Dull RUZ Reduced breath sounds
B Deviated to right Dull RUZ Wheeze
C Deviated to right Resonant RUZ Reduced breath sounds
D Deviated to left Dull RUZ Reduced breath sounds
E Deviated to left Resonant RUZ Wheeze
A
A 68-year-old woman disturbance. She has also noticed that she gets an itchy rash when she gets out of a hot bath. On examination she has a ruddy complexion and a palpable spleen. Her only previous medical history is gout. Initial blood tests reveal a raised packed red cell volume with a raised red cell mass, along with a raised white cell count and thrombocytosis. What is the most likely diagnosis? A Chronic myeloid leukaemia B Lymphoma C Migraine with aura D Polycythaemia rubra vera E Soap allergy
D
A 45-year-old man who is a heavy smoker is recently diagnosed with chronic obstructive pulmonary disease (COPD). He has no documented acute exacerbations in the past.
Which of the following treatment is NOT suitable in the management of COPD in this patient?
A Annual influenza and pneumococcal vaccination B Inhaled corticosteroids
C Short-acting 2-agonist
D Short-acting anti-cholinergic
E Smoking cessation
B
A 35-year-old woman is admitted to hospital with quick-onset shortness of breath. She has a past medical history of asthma. Her observations include a pulse rate 120 bpm, blood pressure 100/72 mmHg, respiratory rate 30/ min and SaO2 88% on room air. On examination, she appears to be drowsy and exhausted. Her chest is quiet on auscultation. Arterial blood gases show: pH 7.35, PaO2 5.2 kPa, PaCO2 4.9 kPa and bicarbonate 24 mmol/L.
Which of the following would NOT be appropriate in the management of this case?
A High-flow oxygen
B High-dose nebulised beta-2 agonists C Intravenous magnesium sulphate
D Leukotriene receptor antagonists
E Steroids
D
A 19-year-old footballer has collapsed on the pitch. His airway is clear and he is brought to the emergency department, where he begins to recover and denies that he has chest pain. He has never had anything like this before.
Which of the following is the most likely diagnosis?
A Carotid stenosis
B Hypertrophic obstructive cardiomyopathy (HOCM) C Myocardial infarction
D Rheumatic fever
E Thyrotoxicosis
B
A 31-year-old man presents to your clinic with a year-long history of itchy red scaly lesions.
Which of the following would make you more likely to diagnose eczema rather than psoriasis?
A Associated nail changes
B History of distal interphalangeal joint pain and swelling C Localised to flexures rather than extensors
D Well-demarcated lesions
E Worsening in winter months
C
Which of the following best describes the MRI findings in multiple sclerosis?
A Cortical grey matter inflammatory lesions
B Longitudinally extensive transverse myelitis (more than three spinal
segments)
C Periventricular white matter lesions matching the clinical picture
D Periventricular white matter lesions not necessarily matching the clinical
picture
E White matter lesions exclusively in the cerebellum and brainstem
D
A 67-year-old man is discharged from hospital following an incision and drainage of a large abdominal wall abscess. He needs someone to help change his wound packing regularly, however he is immobile and lives alone.
Which member of the multidisciplinary team would be most appropriate to help?
A District nurse
B Health visitor
C Occupational therapist D Orthotist
E Social worker
A
A 69-year-old man presents to the emergency department with ongoing chest pain. He has a past medical history of intermittent claudication and hypertension. He is an overweight smoker and heavy drinker of alcohol. On analysing the electrocardiogram (ECG), you notice broad S-waves in the right-hand chest leads, two R-waves per complex in the left-hand chest leads and ST-segment elevation. He asks if he has had a heart attack.
What is the best answer to this question?
A No – but we need to do more tests to find the true cause B No – it’s just right bundle branch block
C No – it’s just angina
D Yes
E I’m not sure; we need to do more tests
E
Which of the following is not a preventable risk factor for coronary artery disease?
A Five cigarettes per day smoking history
B High low-density lipoprotein (LDL) cholesterol levels C Hypertension
D Obesity
E 12 U/week alcohol history
E
A 29-year-old man presents to the emergency department with a 1-week history of non-productive cough, muscle aches, fever, vomiting and diarrhoea. His observations include temperature 38.4°C, pulse rate 105 bpm, blood pressure 110/76 mmHg and respiratory rate 22/min. On
examination, his chest is clear to both auscultation and percussion. A chest X-ray shows bilateral lung basal infiltrates. The blood results show Na+ 128 mmol/L, K+ 4.0 mmol/L, urea 5.9 mmol/L, creatinine 130 mol/L, albumin 26 g/L, ALT 106 IU/L, ALP 230 IU/L.
What is the most likely causative organism?
A Chlamydiapneumoniae
B Mycoplasma pneumoniae
C Legionellapneumophila
D Staphylococcus aureus
E Streptococcus pneumoniae
C
A 60-year-old man presents with a history of recurrent dizzy spells for the past 4 months, which occur daily. The dizzy spells last a few minutes and seem to occur if he moves his head, as a result of which he keeps his head as still as possible. The attacks are not associated with any deafness or tinnitus and a neurological examination is entirely normal. You favour a diagnosis of benign paroxysmal positional vertigo.
Which of the following descriptions of findings on Hallpike’s manoeuvre would confirm this diagnosis?
A Delayed onset (a few seconds) torsional nystagmus on descent facing both sides
B Delayed onset (a few seconds) torsional nystagmus on descent facing one side only
C Immediate torsional nystagmus on descent facing both sides
D Immediate torsional nystagmus on descent facing one side only
E No nystagmus on descent facing either side
B
A 43-year-old man presents to his GP with a 3-month history of recurrent nose bleeds, mucosal bleeding, haemoptysis and recurrent sinusitis. Besides that, he also noticed that he has increasingly become short of breath. On examination, he had a nasal deformity and chest auscultation revealed crackles in the left lower zone. A urine dipstick test showed microscopic haematuria.
Which of the following is the most likely diagnosis?
A Chronic myeloid leukaemia
B Chronic lymphocytic leukaemia
C Churg–Strauss syndrome
D Goodpasture syndrome
E Wegener granulomatosis
E
Following a protracted stay in hospital following a severe chest infection, an 83-year-old man develops bloody diarrhoea. What is the most likely cause? A Adenocarcinoma of the bowel B Clostridium difficile infection C Norovirus infection D Salmonella infection E Shigella infection
B
A 21-year-old student on an internship with The Guardian travel section has recently returned from a backpacking holiday in West Africa. For the last few days he has been having headaches, flu-like symptoms and muscle aches, and now he has started rigoring.
Which investigation should be performed to rule out malaria?
A Blood cultures
B Falciparum antigen dipstick test
C Liver biopsy
D One blood film
E Three thick and thin blood films on consecutive days
E
You are asked to review an electrocardiogram (ECG) in the emergency department. Helpfully, a summary of details is printed at the top as follows: rate 88/min, regular rhythm, axis –20°, PR duration 0.26 seconds (constant), QRS complex 0.08 seconds, QT interval 0.2 seconds. You note that P-waves are only present before each QRS and that the rhythm is regular.
Which of the following would be the best summary?
A First-degree heart block
B Left axis deviation
C Left bundle branch block
D Refuse to summarise until it can be compared with an old ECG
E Ventricular tachycardia
A
A 65-year-old man with a longstanding diagnosis of chronic obstructive pulmonary disease has been reviewed by his GP for deteriorating liver function tests and clinical signs and symptoms of cirrhosis.
What investigation should the GP arrange?
A Alpha-1-antitrypsin serum levels
B Alpha-feto protein levels
C Anti-smooth muscle antibodies
D Gamma GT levels
E Hepatitis screen
A
A 44-year-old woman presents to the emergency department with pain. The pain is epigastric, sharp in nature, worse on lying flat and during inspiration. She has recently suffered a chest infection. She is not a smoker. On examination, she has diffuse inspiratory crepitations. Her oxygen saturation is 98% on room air. Her ECG shows widespread saddle-shaped ST elevation.
Which of the following is the most likely diagnosis?
A Acute pericarditis
B Angina
C Myocardial infarction D Pleurisy
E Pulmonary embolism
A
A 55-year-old overweight pub landlord presents with a several-year history of episodic acute painful joint swelling that started in his left big toe and now affects his knees. Symptoms improve with use of diclofenac. Gout was diagnosed on his first hospital visit, however this now appears recurrent. He developed an acute attack in his left knee 2 days ago.
Which of the following represents the best plan for prophylaxis?
A Keep on long-term diclofenac with gastric protection
B Start allopurinol now with non-steroidal anti-inflammatory drugs
(NSAIDs) cover and increase until his urate is below 300 mol/L
C Start allopurinol at least 2 weeks after the acute attack has settled with
NSAID cover and increase until his urate level is below 300 mol/L
D Switch to long-term colchicine
E Switch to use of depot steroid injections
C
Which of the following conditions does not classically cause hepatomegaly?
A End-stage cirrhosis
B Fatty liver
C Hepatocellular carcinoma D Myeloproliferative disease E Right-sided heart failure
A
Which of the following conditions is associated with sclerosing cholangitis?
A Autoimmune hepatitis
B Coeliac disease
C Irritable bowel syndrome D Pernicious anaemia
E Ulcerative colitis (UC)
E
A 35-year-old homeless man presents to the emergency department in a state of unconsciousness. He was fitting when the ambulance crew got to him 20 minutes ago, and a friend at the scene estimated that he had started fitting “around 15 minutes before”. His friend informed the ambulance crew that he is a known epileptic and you find a pack of phenytoin on him. He looks dishevelled and smells of alcohol. He has a blood pressure of 170/95 mmHg and temperature 37.9°C. On examination there is a quiet systolic murmur, though it is difficult to fully characterise.
Which of the following investigations will be most useful at this stage?
A Computed tomography (CT) of the brain
B Echocardiogram
C Electroencephalogram (EEG)
D Magnetic resonance imaging (MRI) of the brain
E Phenytoin levels
E
A 74-year-old male ex-comedian can no longer perform at smoky open- microphone nights due to shortness of breath. He is coughing up frothy white sputum, which recently has contained a small amount of blood. On examination, his chest demonstrates diffuse crackles on inspiration through which you can just discern a mid-diastolic murmur and a loud first heart sound. His chest X-ray confirms pulmonary oedema.
What is the most likely underlying cause for his symptoms?
A Lower respiratory tract infection
B Mitral stenosis
C Non-small cell carcinoma of the lung D Pulmonary embolism
E Small cell carcinoma of the lung
B
In clinic, a retired 62-year-old man presents with shortness of breath on exertion. You find a collapsing pulse and subsequent echocardiography confirms aortic regurgitation.
Which of the following is NOT associated with aortic regurgitation?
A Ankylosing spondylosis B Aortic dissection
C Marfan syndrome
D Rheumatic fever
E Systemic lupus erythematosus (SLE)
E
A previously fit and well 70-year-old woman has been admitted due to a fractured neck of femur, and she has recently returned to the ward after a cemented hemiarthroplasty. You are bleeped to the ward to see her, as it is noted that she has had only 30 ml urine output in the last 3 hours. She is asleep on the ward, with a patient-controlled analgesic device in situ. Her airway is intact and her respiratory rate is 12/min with normal saturations and good air entry bilaterally. Her pulse is 125 bpm with a blood pressure of 95/68 mmHg and she has delayed capillary refill. She has pale conjunctiva and a temperature of 37.3°C. She has a 12-hourly bag of normal saline running. Her catheter is draining concentrated urine. An abdominal examination is normal.
Which of the following interventions would you try first to increase the urine output?
A Fluid challenge of 500 ml 5% dextrose over 10 minutes
B Fluid challenge of 500 ml normal saline over 10 minutes
C Flush the catheter
D Start antibiotics for presumed sepsis
E Stop the patient-controlled analgesic device
B
A 23-year-old woman comes to see you about her stools, which over the last couple of months have become extremely foul smelling, pale in colour and difficult to flush. This has been associated with vague abdominal pains and a bloating sensation. She has found this very embarrassing as she lives in a shared house. She is normally fit and well. What is the most likely diagnosis? A Chronic pancreatitis B Coeliac disease C Common bile duct obstruction D Cystic fibrosis E Giardia infection
B
An 83-year-old man who was diagnosed as having Parkinson’s disease 3 years ago has been treated with levodopa (L-DOPA). Whilst he initially responded well to therapy, he has started to be increasingly still, and has fallen more in the last 4 months despite no intercurrent illness or change in L-DOPA therapy.
Which is the best management option?
A Add a dopamine agonist (e.g. ropinerole)
B Add a peripheral dopamine antagonist (e.g. domperidone) C Decrease L-DOPA therapy
D Increase L-DOPA therapy
E Stop L-DOPA therapy
A
Which antibody can you expect to see in primary biliary cirrhosis (PBC)? A ANA B ANCA C Anti-mitochondrial antibody D Anti-phospholipid antibodies E Anti-smooth muscle antibodies
C
The left anterior descending coronary artery usually supplies:
A The anterior wall of the left ventricle and the atrio-ventricular node
B The anterior wall of the left ventricle and the inter-ventricular septum
C The anterior wall of the left ventricle, atrio-ventricular node and the
inter-ventricular septum
D The inter-ventricular septum and the inferior part of the left ventricle
E The sino-atrial node, the atrio-ventricular node and the inferior part of
the left ventricle
B
A 42-year-old woman with menorrhagia is complaining of tiredness. The GP does some blood tests, which reveal hypochromic microcytic anaemia, a decreased ferritin level and a raised total iron binding capacity. Platelets were slightly raised.
Which of the following is the best treatment for this anaemia?
A Erythropoietin
B Iron chelators
C Iron supplementation
D Regular transfusion
E Regular venesection
C
In which of the following circumstances should angiotensin-converting enzyme (ACE) inhibitors be avoided where possible?
A Glomerulonephritis
B Lupus nephritis
C Renal artery stenosis
D Systemic sclerosis with renal involvement
E All of the above
C
Which of the following correctly describes Staphylococcus aureus?
A Anaerobic rod
B Gram-negative coccus C Gram-negative rod
D Gram-positive coccus
E Gram-positive rod
D
A 78-year-old retired groundskeeper presents with a 2 cm skin lump on his temple. He is unsure how long it has been there. It appears to have a rolled, shiny edge with telangiectasia and a central ulcerated area.
Which of the following is the most likely diagnosis?
A Actinic keratosis
B Basal cell carcinoma
C Keratoacanthoma
D Malignant melanoma
E Squamous cell carcinoma
B
A 54-year-old woman presents to the emergency department with a 2-month history of intermittent right upper quadrant pain. The pain is sharp in nature and radiates round to the back. On examination there is no jaundice, no hepatomegaly and she is apyrexial. Liver function tests and an amylase are normal. She has no history of recent foreign travel. What is the most likely diagnosis? A Biliary colic B Cholangitis C Hepatitis A D Hepatitis C E Pancreatitis
A
A 23-year-old man presents with a several-months history of lower back pain and stiffness.
Which of the following symptoms would make you think of ankylosing spondylitis (AS) as the diagnosis?
A Asymmetrical tenderness on palpation over the lumbosacral spine
B HLA-DR4 genotype
C Pain present on waking in the early morning
D Scoliosis present on examination
E Worse after heavy lifting
C
A 75-year-old woman in the pre-assessment clinic tells you she has mitral stenosis.
Which of the following is not a sign of mitral stenosis?
A Bifid P-wave
B Diastolic opening snap heart sound C Double impulse apex beat
D Mid-diastolic murmur
E Peripheral cyanosis
C
A 35-year-old man presents with a 2-day history of right-sided facial weakness.Heisotherwisefitandwell.Thereisnopasthistoryofneurological symptoms. There is no history of preceding infection. On examination, the middle ear is normal, the salivary glands are not enlarged, and there are no other cranial nerves affected. The forehead is not spared. Neurological examination of the limbs is unremarkable. Routine investigations are all normal.
Which of the following represents the most reasonable management plan?
A Aspirin, dipyridamole, a statin and an angiotensin-converting enzyme (ACE) inhibitor
B Penicillin-based antibiotic therapy and antiviral therapy
C Steroids
D Steroids and penicillin-based antibiotic therapy
E Steroids, antiviral therapy and eye protection
E